You are on page 1of 98
‘Mircea FIANU + Marius PERIANU = Dumitru SAVULESCU 2) .Orice muni veal este amir rational”, ‘Bnet wn noir my este rational, une muna ni este ime.” ) .Qeice nunc inireg ese pane nat” ©) -Orive numir rafianal este inde net.” P Un nuemae este natura! numa daca nusnarul nu este intceg.” 15, Soeieyinumarul 12 ea a) sui a tre numere natucéle: 5) soma 8 dowd numere introgi din care ual negativ )difereataa oud numere inuewi; dy produsul s dav numere aylonsle: © produsu! 2 dou’ nurnere irajionale; —f) Suma g dou mumere injionste 16, Reprezentati oa sarne de produse inte cittele din baza 10 gi poterle ale iui 10 lurmdtoarele eumere rationale: 739; 5} 0,148; ) 15,345 a) 25,208; ) 218,08 D238), Rezolvare, oj 739= 7-10" +3104 9.0" OUSI4= LES 10! H3tO TIO sau AS41404 Sto A io ie 17. Determinji, in fieeare din situafile urméeoaze, numersle Inregi wt pentry care selajile urmateare reprezinti proporiit adevarate: 1s 2s 4 ? Snel a anit saree: Pon03 #8 3 4 inte 2 si a 18, Setieti cite douf numere rationale cuprinse tntve + 51 4 sub form de i cae ionslecuprinse ine > si ds for a) frstié ordinary; 5) fractii zecimale pericdlice; 0) fiacti zecimeie Fie; 1, Deering nunvesele naturale nenule x sy pete care FOS Hy, 26, Numarole 123,128123; 04142859) ai 7.2081 su serge aus form de aie zecimal, 4) Serio 10- cla de dup viqgul a Rese sumt, 2 Detenminay a 100-9 ed de dup ving este nom 21, Afi est mai mie sud natural neni 2 pes cae facie , 2 9) diel ma tual nena a peru cave facile @, 2 9) reprezints simultan numere naturale 22. a) Determizni numerele naturale pentru care frnetia este reductbil 30-2 '5) Determinati sume eelor mai mci 2011 numerele naturale nenule m penwu care Factia este reductbil 22, Allaji numesetenaturtle n pensrn care leaeile urmftoare sot selene Boe py ane g etl © Tones ” Sia8* Powe oe a dD gory 2a. Fie nominal w= REDE, BLAMED ne Bsik Artiofi ct oe 25, Demonstai c& numerele unstitoare sunt iafionale a) N35 6) V5; fp unde p este un nummar natural prim ot este numsarraional adies exisa fet itedoetibit Rezolvare. «) Presupnnem cd este numar rational adi ca fret ; cau ea, Dedusem ef 3]o., adied astiol inodt f= 2, echivalent ou 3 ce teat =F cchivalent eu 38° =u? sas 1 eta, 2N* -Insamm && 3" =a i oo 3b tet B= 34,4 eH, Decl $= este oti alti, coma Deducem pretupuno Mente: Result ct prosupuneres este fais, ded V5 este mar inion 26, Stabiliji dacd numarv! ¥A este rational in fiecare din urmatoarele cari 3454744199) 9942-14244. +199) 27, Serie clementle moljimslor 4 onl 5 frescos, ocelcn| 3, woe Ze o 2, Serieti elementele mulisnitor (ert: 29. Determinay ciftele a 6, ¢ astiel incl s abs toc relate: a) Gal 3; 8) Tas 19; ©) Sub 13; dy 20h 2365 ©) 09675: 45: fy Babe 198 0 C= {vez] 1523 6 Echivalent, sriem 5 > saw a= by spumer ci numine! weal w este na mare sc egal cu aural eval b gi nok o> b sa, echivalen, spunem ef este me mic sa egal eu a ska b 9 sau «= san >a 2. Grice mums posite este mat mare deeit zer0 yi orice pune mei este Mui uc decét ze, 3. Grige numdr negati este mai mic decat orice aumar povitiy theprietatite reintie! 4e orsine Reflerivittout 4% et, ovieare arf murs ecal Anvisionet it Orieaee at fi numevcle asi b, daca a2 b si BE a mtunci a= 6 ‘Trnitvitedeu: Oricate ae fi numesele a, b gic, decd a> b gi 62-2, atunci ae Rolayis 2 este conparibils cu eperatile de adurare gi inmultire 2 numerelor reale. adie, orieare ar fi nunierele a gb, inogalitaten > b este echivabenté eu: BarerbseweeR: 6) aozbe.Verd; — Gaesbee, Yee. 5. Daca a2h si ez, atunci area bed Dach «2620 si e220 atunci o-e2 bv 20 Partaa intraaga st partea fractionara a vrui numar real Parte intrengd a numsrulai read x notald fs] este cel mai mare nuindy Intres amie sau egal eu x MATEMATICA Clasa a 2 5 é z 5 i = 2 Nurse jx} = 6 [s] se numeste povten flaerionerd a mumrului veal.» exemple. Ts [IR]= I, denorese He 1,532 gh 2 fols3] ©-2 finde 221531 8 1,53) Proprietafi ale parti intragi a unui numér real A. (ex, ya 3, Rotujirea tui seu eroare de cel mult 10° "este r= Exempli 1. Pay 9-1 se obin proxi 10° (atl sp; sprosimaile cu evar de el lt o zee). Fe x= 24, (or! fl0-24,53]=245 le ix pe psa prin cs, ew erowrecel mute eu o aproximnare a Iwi prin lipste ~oapeoximare@ Wt «prin adaos: = 10 N{uo!-24,53141) 24,6 24.83-24 a6 248 7 far 103 0,07 emul com OS $= snamicol 24,8 rotunjeste pe 24,53 eu enoane de ce mult 10" fo 2ecimr 2, se objn aproxsina ale i» eo eroare cet mull exit ew 108 elie spus 21 Awvom 0 flo 21,4) 2. Peme p pcos inate fa esdinalsuelar). Pent 1083-300, 8 gprasimate a oie prin lips ~~ oapronimare a lui.e prin udaos: x, = 10% ({l0"?-321.4] +1} = 10° te 1. Reprezantag) pe ax nomerele ~ 3; 32 ~ 1 25 Os A, hund ea unitate an segment fe tan 2. Luagi ca unitate vin segment de § ery gi reprezentali pe ax murieceles ~1s 0,8. 1 71.2.3 3. Comparayi numerse ay 7 51205 ADIL: gh 0.012 si-0,0021 5 4, Comparati numerale: 251-12 P~O.898} gi ~ 089%, 1 8.6869) $i 9.469). Bj 751 5 1) 2,022 $12,202. 1) 1423 $11,208, 3 -4 yi-t ot b) ay Saye ny 2 oid o 4 wi 5. a) Sctiti fn ordine ereseatoare nunmerele: (4, 6) Ondonasi desereseBtor mumercle: 18; —V2 5 3V3z~ 15 947% 6. figuen de mai jos, sun seprecentte pe oxa mumereiorpunctele O10), 4, sh P asf inet OM= 5, OP= 1, ON=3 x oe oo > 4 Determinaisbscisa fess dine punctele ME A'Sh Ps 5) Caleulagi lunginsle segmentekor [M1 si [Ma 6) Vorificati dxea purctul P este mijlocul sezmentula [149] reogh gi partea fraeyiennr’ @ urmitoarelor numere res 7. Atta partes w Sr: 72; 925): d-48) 9 -AnN3; 3+. ! “ wet: yt 8. Dati exemple de ire} mere irfionale cares abi pacoa Tinton egal eu: 0) aha ons ont 9. Coleulai x] +243} unde ao s212; br34 wae43; Wxr- 14S}: W 2 > x23; Dx doa Wea freemen vio -1 2 = Mircoe FIANU + Marius PERIAR + Pio numnigul @= 125481632 1e) Apresinnai namgnl at cu 0 ereare ce cel mult o sutime pri lipsa 2) Aprosioati numvel rou 0 exoare de cel mult « mine prin aduos. ay 0" 14 Apeoxsinal’ namaeul f= 1000-e eu eroare de cel mull 10° prix lips, Calculati 0 = J279,3843 on patra zecimale exacie gi ape a) yi Ja 2ecim eusmeul a: 2) sriesi pumnul a cu aproxienaie de o ralime prin lipst ¢) serie pumila cu aproximatie de 0 zecime prin adzas; 1d) soviet! parcen nteoagt @ mumcinula Peniro numinal 27,883761, sore o aproximare: ay cu erosre de 10° prin adaos, ow eraare de 10° pein bpst yew erearede (0 prin adaos: «8.03 ergore de 10 psn ips <6) Rowunjiti numa eu 0 eroate egala cu cel mult i | * Cotta ese Ass unde vafoa}-{5]-eol eee tis} t-ts}-G}+t-109)- (2.71}+4.9}- Penis ieeave abe RR, considerim aurerele Jo, daca a2 ja, daca a 23, Ordon eescator amitoneleseevente de numer rae: 0) 3,209.35, a0, N50, NS 2) 12,506, 93, 400, 2050. 15: 6) 31, VF, 188 12G0,1NF; ap 50, 215, 905, 078,100, 15. BNA SBN. -ANT 11g 1-605, “NIT. 58, VE 24, a) Daiexemplu de douk numero reale ei asfel neta > $1 fel < () = (6) =0.7 i 4) Dati exempta de dou aumere seale «gi b astfel ined ta ab <0. Caleulati (2-8) in aeest exe. 28. Fieasi b dou numere eale, a 65 oe 8+ Fhe ST 31. Comparai nueva wo Gills si fivsnv3 + sig ia es 2 vp y ne stree a Bs winds oe ovis WB sid + ~Ji3 9 Jid-3 2-603. Cum rei oN ni fal Rezolvare.« Fis (40) origines i y ponerul Ak CCansteuin paral 0.60" Pri eae, Os? Cu ajutorl conpasuli, detain pe wet puretl 001 sind kak 33, Se considers namercle reale x si.y mai mari sau eyale eu 0. Ariat ct a) Dad vt y=0, alunes 1b) acd w-ys0, atunei ¥=0 sau y=0 24, a) Fic «2B, x2 4, Demonstrati eysliatea x 4r=4 =( S442 by Artal ed dacd s+ ys 4YNoS +6 YP=9. stamel ¢=8 gi r= Ls IMisces FIANU Marius PERIANU + Dumitru SAVULESCU | astfel inet a7 + wees eav. Arai et 35,0) Fie numeree reales, wy Pete oad 17 Se consider tvunghivl ABC de lost BC= [alte be» «| _ [het ASC west ehiater Aratayi ek Urabe re |” |ledaud AAMC est echilawerat 36. Demonstrati cb pent orive nnmers_reale povitive Ae. dk ane toe inegulbaten: (ea eNovil > Yb Yost Cand ue toe 0) Fie x90. Artal ca 2g are Ani vty 4 a) be | ca gotbse Arcata PHS pam arie aie ®o ” Wel Teh Bee e+e 7 ° 38. Determningyi numerele reale a,f.e € pentru eare ace los inegaticate data + Vip? —)2b4 25 VF eH 27 sw ‘omporaji mumerele «si 6 dacit IyaVoet si banal 4vne?: 2) by a= Stands nal gi b= Wat? 02d Probleme de sapte stele 440. Determinsyi partea intreay 4 fiecaruia dinire numerele 4 51 8, unde Lit l LT untewe tr 39. ETCH 43, Pex un nomic eal, Arata a [x Jefred]- [2c] thtennatea tai Hernnte 44,S¢ consider. mun al poss 5 CFTIO" Je respoti x, = 10° (108 xb ip, ei pumini inveg jr gi numente = otmei r= 100" se s1o7 MMRTEMATICA Clase 2 Mircea FIANU + Marius PERIANU - Duca SAVULESCU Tema 1.3 Modulul unui numar real Fie este un nude ea, Numi mob! (Sou weloavee absobyed) a numatnlui seal x, nurse geal sat ||. dent prin se [ Fe poatne 20 xf} 0, pevirn x=0 . <8, pentru x<0 Proprietafile modululi unui nur 4.[i20, orieareartiveR. 2. ]x|=0 da08 si momai does x=0 8. [rl=l-sloricarearfi re A dry S.[o4yfs [ci 4] yl otiemrear fi sy eR [x-ylSjx]o] | oricare ar xy eR. 6.||x[-[yl]]9—p |. orivare ari x.y eR real s]i|y|,orieare ar fh av eR. a. =[x1 oricare ar ve 8. Dact ve 8 si a> 0, alunel |r|0,atunci |rl> a dao gf numat deed x<—a sau x>a 4. Calcul alr By | sPis OI 41-2413: O21) 21285]; PH eT|s wi OPE 2, Stabilig valoaen de adevsr a fieefreia dinwre prepozitile: oO l-S|n-Ss | 23/5235 2 ~)-6.511 ajfeij=[+tifs ef [eV5 [ato 3, Reprevena pe ote w axa numerelo elementele fevtrea dntre mulfinile a Az {reZ}3sel<6h; 2) B={reR| [i= 2}: @ O={ve8||r|=-s} 6813 2 C=[veR| 122}: 4. Determinayi namerele rete x gin © aw |ej28 Pyle Veds @ Bee tl|ed: @ | p79: ep [x= 102 2; DixsSlit OB s By avesinas | xm . Suabilii valoaren de adewae a propoviillor urmtoare: a |-T45]=|-7] 415] bI2s| Mish Sia stsiae siecle oivtenslei ab. sfaSlealSl a r= 73 4b) |v—5]=0; e lavsifett dj |x ©) |x-3] D Ax-3]-1 a) |a-T\ 4] ee : (bp |x yl +[3x42¥-1 ey [Bee 5/53) o) An BI-D pollens a + |x-a]ef ery) D BiyielSety-13 [20 we 8, Dewcminal ourerele re R astel inet a) fixe5y =10 wo foacenae 9, Caleta a} fv sles} ef20e I pentru e<— 6: By ced] 2e—§] |e = ponte «2 3: 0 |etita—¥-[e= a) pemen iss? ofa =35 BB+ (x-2] 10, Determincy valotile lecdscia dintre expresiile urmstoare, in conditite indicate: a} [v3] Ya penteuree bp fB—aF lx, pene x [3:7]: VC 8649 penta xe f-10) gy =4(e=1}: fe 2F pentey vec 14, Doveriioay veree realex eare verified egal a x-3): By ||e—31— o) ||lv-a-3]-3]=5 12.8 aly oe dearlovir a proposifil: _ ~V2y 8) [3.14— 7] 42.16) = 9,02: oh VNR-4) 4-8; & QN3-3) +Visv245) © 18, Deteninali mercer astel inst a ix Ailes 3] £05 Bj [2v+18) {lr-s|-Pevaljeco. MATEMATICA Cisaa Mircea FIANU - Macs PERIANU « Durittu SAVULESCU a 15. 16. 18, 19, 20. a 2. 23 | Fie x. )@R astel nent -1¢y22 i 2: a) Bie sy Rast OM Ftee Wiyai-ly-pisety so Folosind raptol eo +L orice or fi 9.6 ¢, semunstray’ et wb pentus ure a by eljed law Rds OAs 2 1 th Sly e473 249 ef M2 BL 4Ly $I A3e 191230 2 uw loe inggaliatite via8s Arts c& walt exprsilier unntioge mt dep de vatrie rede le ul w 3-50 ele ailelayiowal el; oy Mean Ne ]-Wi elas sess a- Vso f2-se|- olf) er Pontcu Hecare @.iR . considers numerele ois 1 das 2b b, daca ach ashejantl fa daca sb Yiveaoa 024 ath-fo-bl si minted svante of Aratayi et max(a.6) sf mga =1, Alla! valoasen expresiet ot ts 2)+Y9Qy aay ten ay ay ea Bee Be “Lees yi 3 ys4_ Artali et valosrea expresiet ferns fare ses? fie rae + fey sP cste constants KER Fie ye ase rt fc? iv fc2, ante M2 be nenvtngint la drenpta gi mesic sting: sdeschis la stings: (at) = {re R]x> a} nehis la stinga: [as 9} = xe R| x Bah observ Mullimea #: este wn interval nemaeginit: P 1, Seriefi sub fox de interval gi apol reprezentni pe asd multimea m W223; Bred; x65 Maras. -lsxe3; Pp Weved; gy -2svesi Mave? 2, Precizat care dintreunuljiie de mai jos sunt interval: CaWeR dre 25h: ~ ven cc 351: B=[-K1: Ba |velRr-3Sx MIATEMATICA Close 0 itha —1 iecoa FIANU + Marius PERIANU - Dumitra SAVULESCU 3. Subiliti valoarec de adevte a proporititir: a) -2e[-2,40); 3203] OTe S.75 OLE (0,42); o Sebn.-4) B BeC-S40) 4. Scrieti urmtoacele malin de nomere reale sul form’ de intervale Folostad simboluti a fn exemphl dat: {interval deschis la stinga 7, tachis la dreapta 3 se serie (~75 3: Dp interval desis a sténga 0, deschis la dreapta Tse serie w.. 4 6 itervl inchs la stinga ~1 Inc la dreapes +] se see... ¢O interval deschis tastings 4, nemaepinit la drcanta se serie 2) erval nemfrginit 1 stanga, ines Ta dreapta 9 se serie A interval inchs ln stinga ~6, neinacginit a dreapta se seria 5, Stabiliti valearea de adcvar a peopeaitilor: oy He[O;+%6)5 2B) [rN ay (2.3) f-2. 40) 5 (AL I2EQ 6, Stabilifi valoaree de adevér a propozitior ay De(-mal)sau De[Qteah"s H).0€C4D $1 OKI” 6 Fet-24] si 34-29" a 0e(-4,4) sau DEK)": 7, Eteetuat,seiind muliimile sezultate sub fornia de interval 9 OSU: BRICGN, — od 2,G,205 OW ARMIT: SB GIDAG,2); Daa SYM: As a) 44] 0:10) By (ns 20405 9 RABE. £8, Exprimati vasinia mrimilor urmitoare folasind notela cu interval: 1) O vasbie whieste x ani, «$12 4) Pe aulosirad se ponte civcula ex viteza x, cel mult egal 6u 130 kmh 6) no 71 de tonmnd, temperature ¥ a aevaluvariaza incre 2°C (inclusin) $i cel mult 14°C. up Cantata x de detergent dint-o cutie este de 2kg 5% OR eM: D-13}=(0.3] ecotvave a) Dac. ex drs nani Vig une’ wi tres © [0.12 9. Efeswot DELMAS: oan): d@) Sol-33]s od) (ANCE 10, Repcezerat po axe numerelor muljinea nuinerelor alex ea proprettc: arstgire wslsds one? 11, Porind de Ia echivalem | 6868 si numal deck -as.95.@, unde 20", cst untarste mis sti Rom de interval: 4) B=[xeR||3]<3} aR] 2x43] s9}: oO ARO: # (NI) __ f<-s 13. 14, 15, 16. ”. - Pornind de la echivalenka .[al&erdaet si numai deca x saul 2a, unde 02 0%, scr urmarele mulfou ca reuniune ce sou merle a) A={veR| [23] 9) B=(reB] [s|>7}: 9 C={veR| b> 9} of) DalxeR| |e 22} e E={veR| per+3inni}: 9 P={eeR] Bo-l25} re[-2.6]} i 8={reR||*53} 4 Scricti ulm B sub Form de ioterval ') Doterminaticard 4 (umdrad de efemente al mull 9 Bfectuai dB: AB gi AP Se eonsidert mltimea = tx & [x divide 28} 1a} Emumeraticlementele muljimi ») Deserminatt cast. 6) Eleemsay DAZ, Seconsiders mulimile 4=[re Kk Serie sub form de intorvale writen multi 4) roulioea numerelreeale Negev: 2) wines numeizorceale sai miei det 3 ‘) smlkimea rumozeler reae mai mae sau egal eu ~ 25 { rmalfjines numeielor rea rai mar dec ~1 gi mai miei sau gale eu 7 2) limes aamerelor reale pazitive mal mict deedt 9, A) mullimea numerelor Teale mai mici dee 20 gi mai mari sow egale cu JS evil ca interval si puna fs eviden pe cSteo axa a namerelorFcare dintre pullin a d=(veR| -16386}: 9 C=|veR| -S<050); o E={veR| x24}; 9 G={reR| xe2}: <5}; Seve en interval matinle a» d={zeR| [r+i]s3}: a e={ve| A hs ey 3} prolveR| by B=[seR] Pers] @ D={reR) 3s x5} D False] va} ty H=[veR] x23} pia{ren] F » Fe(ver| Pave vif 8) B={reR| [2v-t]e5} reR| pele, oo-| 9 eafrer| MATEMATICA Clase a ro op [onan] hed) = [OAT LINE ep daca -Hehoy 9 G~fre| Irons 06) wuss pa Criviah AUR AAR A BVA AH BOE wind 121, Provizti cel mat nie nui inbeg care apertne inter Fete ‘ * re Serene sePlj|sa} i BF a ra[xtt}s v 19, Precizayi cel mai mie numa Toveg care nu apanine intervals fac ee ee 20, [Determinayi vel pia) mace nunwirimceg care apacine interyalulul 21. Determinati cet mai mare numar inteeg eare mu aparfine intervaluli J. dad 48 oy tz fek| plc Peet: dal-Behs od we eR |sv0-7i4} 51 Bfrerl 2s 22, Hfecuti eet my 2syids 9 A6IBs fp anlver| [asin sigefver i+ BBS; AMOI: f-2,8}V(0.217 l @ AO[S WSF DS] -43] 28, Delezminaji »),7= Bi stiind oa 23, Fie « gi fhdoud numere wale eu <9. Dack #=fash]. E= [aud = (0,0) sa ay {ceR| [aisd}e fre Rferte taal: 7 =Caib) numical (1) 2foe—b se numegie fongineer tervoflt mrgint by fee | [3055 Jefe Rheem) Calculag lungimea HieeSeuia dintre intervals 29, Determinat, in flecare eae, imtervalul Feare indeplineste simaltsa conde 45 opin o(59) 9[-$-9) fueastetean 8 foe sin(S 24. Seriefi Gecare dintre urmitaarele multimi sub forma de aterval. Caleulail BY ELAS OA] $i P-L =H Aungimea Flecarui inter! byinut fo Pups =[-S3] 1532005 a) A=fvelR| |fsio}s 6) ay 1V238)> 2:6] $1 F128) = (88) 5s.) FUP T= (Te) 93 ctl a(R: oc {5 ni astSeahiao DAS CBI= C2 POC RE|= SH) eo) b= {reR| 2vedeths ol}: 9 P=fveR] 3y—1ei-5, Df. 30, Determisati numerete tury agi 8 wile . Rezolvare. &) Inmektvd eu 2 ficcare mensbeu 9! dbl inept s¢adundnd ape 5. wp ubON= Bhs HLebrE=t-3.-2h oF ees olWingsn -O51=-5<6=9-16 eS deci € =| -1.1I gt ME} |N Dis IZ BANK 2 MAE 23)5 Plead 25, [Determinati Cun ¥ fn fieeare dist cazarile Rezolvare, a) Dewarece 3e (usb), rezultd <3 gi 2 gy X=foal| ess} dy Xafeek sheds a) [RA )[eR=2s—lalis 3A] BPA -BABL A NI) 2 a ve{vez]-a FIANU* Marius PERIAND » Durst a) pumbral a= natin interval | dy rua b= : ‘ Lina 01a - naar) = a a8 wee 33, Se considert intervalele mrginite F si J astieh meat /UJ este interval Detsrminativaloarca de adevdr a Rectreia dine propoziytile a CONS MAY: BES) eHe cen interval {sau reuniune de itervale) mele: fre Rj fos]s2}s B) B=(xeR| [av] >5}s Ri {1}: 4 p-f ren sSfoa} pe ={xer lrodle dl 35. Series; muljimite 53 B ea intervale gi efeceuali ANB, ALB, BY @ A={veB| |v-2]>1} si B=freR] [sl Demansirafi ch (AES 2. 40. Numierele reale ai B gi © verified relation a? +6? 46? + 2a—d5 6: Demanstnics @we[-AS]: HELA; ee l-28] Iindicatle, 4) Adnan) Tn sicmbii sama P4248 gh obtinem: (uF HG=2y 46c- 39 225-8 Reales (e=1ESS* sau la Il sS. Continua! sri an a2, a3, (rp) (tpi (rp) (rp) tpl el {aph (apt ah Probleme de gapte stele In intervalul [0.1] se considerd numerele jrationale 0.044.004 64» Atttati eo 1 fea det 0-542) ame whe da, exit ie astfet in ies Se epnsiderd intervalele a) Deverminili o.be% ase neds [AL O% By Aritati ch 15.4 # pentny orice alegere a numetelor reale a $i e341) gh Pent eaten au 21 scone ines f=] 4) Deveeminati multimile 4,08, fy AN, LN 2b) Demonstrstie@ J, < 1,.,. pantry orice 21 TESTE DE EVALUARE 5 Testul 1 1. Se considera smultimea: 4 seFireo. Ph 4) Seri clmentete muti AQ by Reproventei pe axd numezele raonate din mullimea 4. ©) Ontonaficeseitormumerete din mltimen 4 Seviegicleriemsle muljimit B= {xe |-x« 4} 6) Seriej eementele onshinii C= {xe 4]- ve 4} {reR|l (Serie sb farm de interval mone 4) Deternina ned MEO 2) ©) Adin 8. orcare ar fi @ © R , does ve MF, atinei —o 1d) Determin}i eel nai mare numr real poritiv b cu proprieiales c2, pent 2. Se considers muhimea Mf 4 i asl orice ¥2(-1,0)9 (0) reat od +e 8¢ NOTA. Timp de luctu 50 minute, Se aconta | punet din ofici : & Testul 2 3 4. Se considers namical g =-2,(24. 5 (spl a) Serieytumarul g sub Forma unei acpi iedutibile Fae, beRY g * 3 DMirceo PIANU = Marius PERIANU + Dumitru SAVULESCU inet fp) inp) Hp) op) op) ap) op) 4) Deteinina pate cea cr Calcatatt [g] +[-u] -unde 40 eeprezina pase dare a numa dt Cally paren frvesionaed a nuunarutai nurndrul 3 eu enaarea a. @ 10° 3 ep Ata} utr gsapmesiimeaz Deg 2. Seeansidera muljimea f° — B {4 aes 42) Calvalati suma elementelor otis 5) Doteminali eumarel masim de elemente care pot ti eliminate din rualjumea fasted ineét summa numerelor stimase si tie egala eu 23, 6) Determinali numercle x « F care pot fi reprezentite de lraetit zecimale Linite 0 Giasiti_ mumerele din nmultimea F care sunt pitiate de numer NOTA, Timp de lucru 50 mina, Se acord | punet din otic. op) op) (pr fpr (apt apr (pr api fpr Testul 3 1. $¢ considera numanu] «= V3 44) Devermninati oumfrul natural meu propsietsien c& cereal 4) Dezerminati primete dou’ zecimale ale aural st. ) Scrieti cel mai miv numés inereg din wnilimes 4 eR lls] seh Arita ob Bac 4) Determinayinumeree rafionate.x pentsy care y= 0° 2.So considera multimen Af =firiaijve M38as8} a) Arinaté 00, oricase ay FEAF, numrul 9 exe un mamae MAIER ivizibil cu LO, by Dali exemple de dows mumnece din mulfinen Sf @ elror suma este ppateatul un number easional povitiv. nota eu ©) Aralali ci ae AP d) Fie eM, Se stie of sumarut & aproximenca numdcul real y cu eroane col mult eyatt ew 10°), Asdtati c& numéout 4°! aproximenza momieal ot cou oroore cel mult egald'eu 107 NOTA. Timp de lucrs 80 minute, Se acord® | une din off (sph (5p) 2. ap) 3. Fis pentru portofoliuil individual Numele gi prenumele: Casa a Vlka Jemanr.L.t: Muttimidenumere reate. Compararea numerelor reale, Modulul unui numar real. Intervale denumere reale Completati pe fisa spasille punctate cu rSspunsul corect. 3 ) Dinute oumercte © gi 32, ouneel sree este di : 524. Cel mst ne nuns ciy nnn by Fle multimea 1 = [ce esie agal eu co} Moctulu! nutrulat NB ese Pentru fiecare dintre enunturile urmatoare, dacd enuntu este adevdrat, incerculfi litera A. In caz contrar incercui litera F. 4) 37 este numa ieational. AF i 2 ouaimore este dR ga 8) Dine numer go doce Incercuiti réspunsul corect la fiecare dintre urmatoarele exerciti Dintre cele patru varlante de raspuns, scrise In fiecare cevinta [30) AF doar una este corectd. 6) Su metal intreucontinatein interval [-2.3] este egal ew A 8.0 ca bet [ee R) -35.x-1 sf este cgali cw: A. (2) 8. {-2:l) ) Ascend in oning creseatuare vlemewtele mulyinil se objte erdinea: BNE 8.4, 305: 2N5 {5:1:3:0) - Cem mai sh diferent ire dou CER d Fie mulgimea stemente ale mulyimii C este egal cx AS a8 c MATEMATICA Clasa allo —1 iccea FIANU + Majius PERIARIU + Dumitru SAVULESCU La problemote 4315 scriet pe fia de evaluarerezolvérite complete. 125p) 4 Sodas mile A= fv eR] Bv~i]ss}st e={r= Caleulath ALB ACB, An BB NOTA Timp de vers 50 minvie, Se seurds I punet din ofc: | Tema 1.5 Operatii cu numere reale Adlunatea ese opentia prin ence cvcdreiperechi de mamere ele og # social in mama ea, nat e+. yum sma numer a 3 Propritajile oeratiei de adunare 1. Adunarea este aoclatd: (a+ 5)+e= a+ (8 +6), oieare ar a, 8.¢ 2, Numan real Qeste element nents + O= 04a a, erlearear eR, 3. Orieo nurmir real @ are un opus nova! ~a cu propeletatea: a+ (- a) = (-a) + 4, Adunarea este comututisd: a + $a, oricare arf a, be. Scaderea este operatia prin care oriearel perechi de numere reale w $1 D i se asceiaz’ un numér real, total a~b , numit diferente numerctor a gi. Difesenja mumerelor reale a gi b se defineste prin a-b—<+(-8) Inmultirea esis operatia pein care, oricdvei perechi de aumere ceale a gi B 1 se asoeis2iun nuns ree, nolat @-2 sau ad, rumit prodetd sumerelor a si & Proprietatile operatiet de inmultire 1. dnimulgires este asociattvd: (a:b) -e= a+ (hc), oricare ar fi a,b,c 2 wumarut real | este element new: @ oa, oricarearfiaeR 3. Oriee momar real a4 0 are in favers: o cxpropiates 0-1 -aergerearB abe 5. inmalven este ase it por ev adunrea 83 athe 8 Fnowattiven este comuativds a b= leree numerelor real bare, oricare arf a,b,c eR. Innpartirea este cperatis prin care oricicel perechii de numere eale $i, b 20. ise asociaz’t un numir real potat { , mumit edd mmerelor @ si, definit prin 2 =r"! real pot &, mumiteie mere a5 b, defini sin Ridicaroa la puters eu exponent numarintrag, Fie ¢ Rim & NY. Definim a" 4 si, pentru 20, a = hair ‘Spunem ca a" este puterea # a numacului real a. [m serierea a”, numarul ot se numeste face purer, lar numndeul 2 se numeste exponen puter Observatie, Penteu orice @= avem al =a ,iardaca a #0, defiaim OF eso erate reson eguli de caleul cu puter Tala? =a" * evieare ar ac B® si oricare or fi mae F 2. asa" 2a" oricare ar fi a € RY sioricare ar fi mneZ MARTEMATICA Clase o¥IIla —1 | \Mircas FIANU + Marius PERIANU » Dumitru SAVULESCU. 3. (a) =a" poieare ar we gi orieare ar fh mare B. (u:8)" 0.0", rea arti oe RY siovientear ore 5. (4) = Sosa aoe smear wee Rdacina pétratd unui num rel 020 este mail wal 4 P2U, co propriettea =a, Netins r= Regulide cafe radical ae ada Bave-Vi penn a20,b20. Consent: (a)! =v? neh la |, ortcare arti rel pen a2 091624 viel 3 evo Consens ( fy - cache 20, 620 B, deck a <0, 626 A, Inteoduceres fueteriloe sub radicals a5 lf s.Sconere hetrlorde sub adzal WaT =e petr oie # 6 #26 Medii. Dact a sid sunt numene resla, «Sh, atunei orice numbe m= [a6] este ‘a medie a namecelor a yb ash 1, Media aritmetica a numerelor ogi. este numarut geat m, Media aviemetic& a mumerelor reale 2, Media aritmetics ponderats. souby Sunt numere resle poritive, Fle py. Ps Media aritmetivg pondenatd a numerelor 3.9. fig 6 ponderile Py. PrDy © ‘este numaeul real ,, = SUE Yale Fn saa aE Et Pa Observatie, Daci pj = a swimet modia aritmeticd ponderata s 1, coincide eu medi lor aritmetic. smumerelor 9. 3, Media geometrica a numerelor reale pozitive ¢ sib este numivul mi, 4. Caleul : a TES » 54> @ 2s{-12); 9 94(- 9403) ees, 9 -Bs0; wx nabs: pe-($ed)s oncayelits)-2]: 0f3-b}-a4s) oi): BNE m NE+28; mli-4}-(-4) 2. Cale ove Vb, pe oh a8, p28 .2=-N5; 4, Caleulai: DxaltV3,y=-2N3, = 2-8 wy 56s wants @ BI}: a 3(-V8)s AB -s 36 J ae "To 4 asia), Babe a (XE) (Ws » 25, yo, MATEMATICA Cass 9 (ince FIANU Marius PERIANU + Dumiteu SAVULESCU 6. Bizet t st es 9 Bin a [Nien 7. Beta by {-2V3)-(3V2) 4-6); ao W5{-5-sV5 =2); 2) (NESE) (5423): p (207i) (VE -vF +2) 8. Aritati cA nuinrul x este inversul numérului yin wentoarele situa BBB gi pe V3+VEy a) x=V5-2 gi yo V542 oF gE ye Niel gi enol 2. Caleta at ays oO CON; do; (2) wis GQ 17 Sore! a odin erescatoare numrele oy (SB) 2) Aya) (NB)s 9 (v2)! (BSP SSP O88 SP 4. olosind algo de extagen a Adie pcate, deternsinal rimete dova zecimate ale numerctor: a) Jil; Vids; o) VSNI2; ) JEST 12, Scoateifaiow de sub radical (a,d-€ R ) a IPs Fs a lies o VeP 101s 4.4, Sconiyi factor de sub radical (o.b€ a Si2s 8) 20, aves ON: eS: Vi: g 02005 1) STOW: aR. p88 wy lina® 9 S507 15, Scoatei isto de adia fap eae ira 0) V+ WT 9 NENG AS; 6 NR NR 0) VN «0; 1) “Wii 24700 + Jaa 16, introduce} fictoral sub radicsh ans, y-ail iW ae 113; PavB.er0, ans (xco% NE; eR 17. Comparati numerle 9 Ks HATE NA: . geil, pra gion S39 ab; NB-6 si 18. Determinati primele dous zevimiale exeote ale numereler: a8 yt 9 6 Os 19, Coloulg tia astmetis 9 numerslor: 010 3 By 22 ot si 10 $36: was 2ks gt si 2-8 si teN3; © WI-4 gi WI+4: 20. Calewlati media gecmetied a numerslie a gi wade: a) a=133i b= S25 Band si bu 45v5: ) 0222 gi b=RSR; @ a=3(V5-1) 9 b= 3(v5 #1) we Die 21.) Soria nurnarul J ca smb intre un namde ragional gi up nude itational 1) Seviesi nurcul 2 ea sum de dows nunnere irafionale. 22.) Dai un exeinplu de dowd mumere irationale a 41 & cu proprietatea of iamerele a + 8 gl ah sunt sirsulten sumece ragionsle by Dutt wn exerrpht de dovd namere irstionale pozitive a gi b ew proprieustea & mumerels w+ 5 yi ab sunt simultan numere rafionale, MATEMATICA Clasa 2 Vika —1 | ‘Mircea FIANU + Marius PERIANU « Dumitru SAVULESCU. 23, Medic aritmetie8 « nurmetelor a gid este 4. Deternninayi nucnarl daca a=; a=2S; a> Da 24, Media goometsich a nureretor« ib este 6 Detertinayi numral dack Bet Bo=KS — dandks a o-2We 25, Determinayi numerele #eB fn unttoarele exe + 2 gases; bp 2 "RS ayi5 15 rz oi MS o Vid <8; x-fi) =) eb Vist = JR DACs 26, Caleuay 2) SKE HES IS; 8) 15(-3)[25:6-5) +13} 9 (2p -P 42; A B-2-F 417 17), 2 [oar] =1695 f 811118 =3333" 27, Caley 2) 175-2430, 1b) 3428:10045,72; 6) 34:10-5-2,2418,65 A) 0.25}~3,508) ) NCA) + 10-34, 268) fH 0,1-[0,1? -0.1-," +0,099)] ool akalts) . Ereetwatis o W5=28, wien 6 IIo -F: 6 Di +308 9) N37 #2-5137 «10: DB-NTHAG @) Wa+7-7-); iy (5-34 365 8) 30, Best 1 LNB, y NE -08il: 9 (GF ~2). (64507); o) (2530 -5¥38):(-5J/5) wy (-s0vé):4-s2}; P (35010 +2910) :(-v 100) > sunete . o) 8 (HF LBP HS) Ua) 4 Ys ork [AF NEE TO din urmatoarcle proporiti sunt adevirate: 32, Seabilit 9 S525 5} su —-Wi5 5 ay 307% (86) = 6 eT) VB} + VEIT -VB Jats (NP -20) 23. Caleulat OIE EAE NEE os (nb) e+ SFr AD ENTEBBE ob Fia02 [7-H )-2)} 2 Wi BBS BI a (9 ASA» 6-203) -2NEa 72) oF 80 8-8)-A5 ty (0 —3)( 10+ 3) + (su —2)( sv +2)+(1= 8) 427 34. Fie mumiml a= yi J8) + (V5—1) +5 VE VERIO Determinati sumerele intregi b, esi aundrul natural d, stiind ed w= htodd al: je gine 35. Intraduceyi factor sub radical: 2A: (3) 2,00; poriae 136, Scomtyifatoi de sul radia waar <0: 2 f29-(587) er oy 37, Sconi Ferri de sub radial ape enleula a) Vide ~VI5; 4+ 595 VT, 38. Blectuayi i WG (JT STD = AHI0)- SITS; 835 (NB VIB): yh oy hac IF we 20 eR 60. e+ 80-5 MATEMATICA Clase 8 Villa —1 08 (SRE AR + S12-(VITR + VIR) ooldF-310|+ 7+ V6 WB 2.1]: 1) 62 4 (3075 + VAR} ( J588 — MB VT2R) a) N25 +3 4508P ee > Calls 46. Caleulsi area aritmeties a numerelor - ay {8 48). [S/R 92 = JID. SRT) ap a= (TT 3)(03 + VI) VS — VA) gi b= (i +8) 3 VEINS + VEE (Te) VB TRAE 8) a= (ABE VE) i= (16 BHD) . } 47, Cauley nesia geomettcd a numerclor: o [sv 18/5 }(6v3 + taV2)-6]:9 v6: = (VEE EVE BIS VE) SET + EB) o[ P25 (SAD + 496)-2/735. AF] {v5 + vEZO) be (V5 HIM VWI) SF + SINS — VP (027 — V2) 49 Efectust cateuele 49. Cates a) Tn VT ibe Srl Ties a) 2+ JI1S3 ~JaB (VB - VBE): 24 0 VBS fee fia 08 6 -VaA 19[ SATE {B+ 207) 10) (VE +1) VRE ; 49, Arita chs a) 3-2V2=(\-V3)) gi 1t-6/2=(3-V7s 10 -2002-[ 2 ~4{ BV] 32 9 (6 +2 ew set 155+ SRB. VS) (FRB INH) 213 week [SB -5- (9B NB VY) 9 oii 7 50, Comparafi numerele a s1 8 daet: a2 WWF) NE) ART +4f3 V7) si b= VR eee 2dF tavlb-olF ala hie aa ot: su hue oy felons —f-fis-a on AY. Artal cl ani Ba 2b; MS fii -\FTT— So; o foi Wi ere oak Gon IAS eae 52, Arita . : \ sae — hora : 23034043 99-100 2 bck. bom inne 45, Daca 1 ©H, determing selorile posibile ale expresie’ 42, Caleulati suma: § = 53. Se consider numerele reale y sip. tind ey y > 2, anita (ceo FIANU = Marius PERIANU - Dumitru SAVULESCU ooisy eet ap eter Leen 2 BIMBO TD gOS expresis 6(x5)) = ebony? dyed — YP ny adler ste constant 44. Calculap! x, unde ‘54, Determinati numerele reale x, y si z care satisfac inegalitaten’ ANTE EES) SE aise d+ fy ay 350 1052! ~22 45, Stabiliji cave dintre wemdtoarele propozitt sunt adevarate: 55.Catetele sie ale triunghiului drepiunghic ABC. satistac relatia 0) AF GF + eB AB 10 FEba S14 fe —2Vide+30 $5. Caleulay aia tcunyghintut » INS-MTi-s(ViT+ Bewv@ MATEMATICA Class 2 Vibe —1 ‘Mircea FIANU + 2arius PERIANU + Dunit SAVULESCU 56, Detemvinat nual natural Find a Yee eH s =10. 1 ee Sia Fea Bootes 58. Aritati ci a) VBS On Bb) V+ S434 5 4434547 +e Ves eS + 42015 EN Probleme de sapte stele so. Cateulag [64248424 478 fide ed Sess aa as 80. Coleulati 6), Culeulayé 62. Cateulatt T2324 345° t 63. w) Demonstrati Gay ©) Calcalati sume: safety fbed Tema % 3? Rationalizarea aumnitoriloc Rationatizarea numitorilordeforma ay sau owes ta NO Prin regfonufizares eupnitonulii un report inglegern elinninares radical de la mamnitorul rsportlu respect, Pr & medion waloares mpi Be ol 1. Dach ace Re si B> 0, anne! eS ole ald ne ob a.Dacd cud ER si BC, ne a alt on 3 Ng dla} ye Cea unde ae VR observa, Expresite de tipul «+B gt a Vi sunt wipe conte unde az vi: * 4. Retionalizati aumitorii urmatearelor rapourte OF on 8 B og om oe oe Dag aps 2, Determinai rine doudaecimale exace ale numereir: oe bps ose 3, Rafionslizati mumitorii ermatoarelor inpoarte MATEMATICA Class 2 © | Mircea FLANU Marius PERIANU » Dumitr SAVULESCU ™ : VBS Caleulath Caleta a (-2y's2? 0 (an) (BeaY"s & (i) +(8): oy (Vi JAY of ay" Pent numstele a=4V5 i 4= aE, saul @ ot, bab: gy Me ab 7. Rotionalizati numeri: Yh OT WE Ose ae we 8, Caleatay: ae e = Caleulati de fiseare dats sind ca bei 4) aa i b= V7 12, a 15, 16. 1. 15 - Catewlat xs via ‘aa Calculalé media aritmetics 2 puerto: oo a) 13 JF +6 si — 3)10+ 205,345, 25-67 gi OV +7 Media acitmetics numerclor a, b, si ¢ este egal cu 7Y3, Dock a= -8+6V3 5i 3 dots unl gi media anticas numerelor 5d culaji media geomnetricd « numeretor a) Via si V6: by 13-65 gi 134-89 GME 4 EE says nen SE Media geomaich 4 nunerelr agi b ete egald ou 48. Duct «= 7-305 eterna ude media siietin a numerlor Deterninayi nundol ren a) (150) VT) = 3d +899 fii i ) JF sos nmsrrnt —JFF8) | este namie mere" ia) (an iB) 1 1 vat ai +3ya an Calealan (F-55205): ‘by Devertinali abe en proprietatea: axfS—biF = 955-1201 — {712i & \erernst Clasaa Vila —1 J ‘Mircea FIANU + Marius PERIANU » Curniteu SAVULESCU 2o, clea y MEM AB wk 21. Caleulayi valotilecr-b cer:h: (a 8)", unde ag sunt qumere renle date prin EAB filer Bao si b=[5vB4vi0- 38) L50. Soa ~ Ver) 22, Ardtai_ of, peniey orcare mum real Barter tevéart 2 ay a, 20, valoaree expresici ete eenstar 19485 — Y8+ 2015 + 23. Allati numerele ae Z pentru cane Probleme de sapte stele 1 1 1 fron’ ake ral foe 28. Dotwrminati » €NI* astfal Inet wt wibs Loe eyalitatea: 1 1 1 - 24, Caleulogisumn S: \ 1 BB ini nv 25 Sera) onmntgri Ca me st G8 BK ei = “hr Ba” “her by S= aH a ne — yy © STR NEeME Miva ndieisriga & TESTE DE EVALUARE o Testul 1 [ip] 1. Scrieti mumerele irarionate din malipmes alin NG a) 1-03: 34} [tpt 2. Caleutagi media aritmeties a numeretor: 21; 23 sh 6-23. (1ph 3. Scrieti inverse! numirului ~ (ap) 4. Ardtafi od numdrul x =|? 50] J50 +91 este numar natural (ap) 5. Rezelvayi in # inecuatia [Se—ths 9 fesrl 3% z 2 {1p 7. Stoii vatoarea de adovar a proposie @ se(hTINi 8) 3a (1p) 8. Compara numercte S45 si 5 {1p} 9 Stabilit daed eumira 3.5-[W3~ dl V5 (ip) 6. Caleulagi {8')! I-21, 10) WN.7} sie efonal NOTA, Timp de hueru 50 minute, Se cord | punet dia oficiu, Testul2 (ip) 1. Determinag primele doud zecimate ale nmmarubii 37. (ip) 2. Secivff opusul numiieutui 5-V7 {1p} 3. Inieoducel factorul sub radical (1p) 4. Culealayis (3°) 67 +(-3} (4p) 5. Resowvayi tn R ecuaper flare 3? (1p) 6. Steet: gem Ae: AMOS), eT 91 7c: SEE a i {4p} 8. Caleulag medi avitmetica 5! media poometeet » muimerelr: a est 9 hea 3 (api 9. Calevlagi: 2.418) +0,04-10° (277 +] ~3) + g NOTA. Timp de lucru 30 minute, Se seaeda | puset di 2 a7 rus PERIANU + Dumitru SAVULESCU Mircea FIANU + Testul 3 181 4. Seviet eonjngtel meals SVT Mp) 2. Catculati media yoometrica a numerelor 6+-¥iT si 6-11 8 a).9+8{ 4) H- Sia) a+E (A) op) 3. Rezolvati ccumtia: id —3. Op) 6, Caleulat @L-23UIT) —B CRSINZ: {ph 7. Detemnat celna ne mnt neg din ml dap) 8. Catena: 27 0,3081-10-[2-v5|-VB41b {1p 9. Fe mural a= 13548, 4¢28), Rotujig aval la ordinuls «ssi sy lor 6 recor de rim np) 4. Calewtan = 1.0,1.2,3) 1h 8] freB| ( Marivs PERIANU « Dura Tema 1.9 Descompunerea in factori Descompunerea in factoté a uel expresii algebrice consta in ansformanea expresiai intra prvcire de dott seu mai multe expresiialgebrice Metode de descompunere in factori 1, Seoaterea factorului comun se bazew/i pe propriciatea de distibutivitwe 2 operatici de Tamultire in raport cu adunarea’seiderea numetelor real 2. Utilizarea formulelor de calcul prescurtat Go bbe! + 2ab-+ 200+ them lathteys (uzb)(" rab +5") Pep 1.9.1, Metoda factorului comun * 1. Polosind factorul com, ealeulat: fa) 2S-374+63-28; hy 45-32-32-1045; o) SP -56-57 IP 78-77; e997 98-99-98 f) 89° =88-89" — 89-90 2. Seowteti eetor eomun rere By Sx+5y5 aca; a) ma? +ao® xa: o) px + ps pas Pp ase yyrbxe eds 9) Bln Gort Be Bi 4 2a—V2b: Bima ana bos He oes 2) 20 — 400? +600 mda Dae +80? 5p Sadan, 7st de, 3. Seoateti factor comun: a) ~4x-8yy By Ax? ras; o) Bx 26x7; 4) 9278? ¥30%b e) 3x 9x 4689; DUE Onn) x(a) S416 4320 Aye -5y oP eDe: B fixe by SFtet SAE 23 -5V2a mm) 130-300 ny Vay 228"; 0) Rds? +0.6e5, Mat) leet) 5 Preis 3 3 we 4, Se consider’ mumerele aural a, 8 gi: Calcul P= ab +Aesea, sliind et gbatsecn!; Bl e=lheb Jarhhes 5, Sevatf actor comin ; a) Sa! + 45a°7 -3ax 8) Bx Boe? ASN op Mia! bin 65 + BD (ur ea}eXuine n)-22 lore 6. a) Ariat ca numiryl wb+ ba se divide ou TI, (By Artingi el numaeul Gbe + bea + catb se divide cu 37. 7. Scoateyi factor eomon: a) Hath)—xe+b) B} cen tay btn say galery Plea): 1) Anko} 8. Desvompunat in factor: a) ake dyeten Xess) alu tb)~ (2a 1Ke hs ey xl3ateapeaetzese?}, ——@ (21-3)! -Sa(20~3} RK 9, Descempuneti i ato 0) he #1) ann? ) Ua-xpeaex a5 a (1-2 eP = 1 gy Witarhtey-dMarbscyestori+e) 10, Art ob ovsare mur rep, nunc Mase oma Tne, ade (T+ 3)~6(3+ 7a} ay kD BA 2) by Ul eastaple rari ane Sn 14. Arta cf, ericare or 6 marsh trie 2, rural W ete pirat perfect unde ay N= Sirf88-D 501; by N= 2aldeeip +t) Probleme de sapte stele 12, Determinaji nurnerele reale « cure verifica egalitatile deta 2VS +355 by xt 4; gy of2-n5=1; Gx 3-WI. 13, @) Se considers numerele rationale «, b, ¢ sic Deck aJ2 + a3 = eZ +dV3, arttei cd we gi ha 4) Determinati numecele rationale «gif sind ci av2 + 2% +A - 208 MATEMATICA Caso 9 nL So 109 20. Caleulati rape: GUS AF 196-98. AHO, Aw) 19.54 27, Determninati valouren expres a) 0 tine of ey By wey stitod ea ao sist Flagi eye 22, Descompancti exresileurmoore in prods de eel putin wei fates ao ot—bs: Bal: 6-3; a) Blom ee Bly" fp UGot 9: g) Bla*=25; fp Blin! 16 23, w) Verificat dacd numa p= 117° —74°este numa prim. 4) Determinatl numerele naturale @ pent care numarul ¢= rt —Leste mute pr 24, Descompinel in prods de fio: a) (x42) 95 6) (22-1) =x? (3x42) 250: B(8e-3) -(2c45). ei[4yatP (se -axfiy ax’ {0 ~sr)° 25. Deseampuneti in produs de factor, folosind formula WB ee 1Qob Hoe rhe =(ar bey 496° bons 2804 14m: Bie +2a+ 18x Bers Dio +3y" 9) ¥ tal 41 2ur42— 2a hy 40 43° a Ox Atee= 20 26. Descomputeit in facta. folosind formule: vale Bay" by (sty) sow a? Baty Sap Wal +30 43a 45%; bY HRN Beals Gp AY HIB 6x4 MATEMATICA Clesa a Vilks —1 ‘Mircea FIANY + Marius PERIANU - Durty SAVULESCU 27. Deseompuiney tn faci @) Bo" 2B 4 Gab! —b, By 4B y Ly" — 12597 £15 y 4 1S + ) ~Syn? 430101000; OS erty t ize a8: Dae 1S" y + Fax 8 12S" ee 28. a) Atdiali cd 6 ~5' 44 -¥ 42° ol! = 64544434241, 4) Caloulais § =1° -2° 43°48 +... 99! —1008 29. Aritajé c& urmaiowele expresii sunt patraiele unor numere reale, orieare ar ‘nunéral real x ap a(n rs 3)+ 1: Bp baer NTE ae YGF Bee De dda atresia? 45x 649. 30. Ardtaficd x(6+ 14 2Kx+3)+120, orieare arf pute) real: 31. Sting od Payee 42(-2 rot om) cael [ery | 32. Deseompumnet in factor, flosind formulele oh Hae eer y') yews Darads aves @ P48; MP1; Pi2SP-B: gh mb sats By tat 33, Descompuneti in feetor @ P41 y+48ey" +649": 6) 100 33000456300" 13314"; GE Hoty HH2aty By") ala’ —[Sas'y + TSan°y? 125)" Saye erp") sau 2 Probleme de sapte stele 34, Descommpunet in Faro a) + 6b 412d" +865 by Bete Ly 6) 1280 4757 y+ 1S yy) BP 3abP het aes 1S 15s BS ie 3 3 \ te eee Be PS ab atyty Dey yt 7 4 - p zo” “35 125° 8) 009" 15008 Bata? 35. Arita c&, pentru oviare numienacural » 2 2, num este comps, unde a) Naw enels New tad Indicate Nani ee neler (a lem onclan'(n- lee enehe 4.9.3. Descompunerea in factori folosind metode combinate * 36, Calculalj valoarea expresie: Wa 4b" 4202040), sind c& + by ahah? -Uabtand}, sind oh a hand. stiinded tba gi a8) Pte +2uh tact be) Hasbro), shiind od a+b+e a 37, Descompune in produs de factor araary De sdtede: xt Bes di eI einen, Pavnx, g) 8 -T6ey i BAN" se divide eu 38. a) Aritajic8,oriesre arf numnirul 2. nominal a =" 1b) Artal ch, oriense ar | nusmaral 2% :numarul Ny —w ve divite cn i 39, Descompuneyi, gruptrd convenabi) teemeni a) olx+y)eber bys Bp Us Tes NG») o) Bx46y+im + 2my + A Umm? 415-509? 5 2 PT bates te Bde etme ee 440, Scrieji eapresiile ca diferente de pateste si apoi descompunel in factor ge asseda2s bt 44-5 odes dP -24-3; 2) 9x4 46x35; f) 25¢° +20e-21 wg s-20r4 hy 8 -Re-9; st +205 498 piexndy Wess Dt nx-6 my) x! ~5x+65 nj 8413x430 UX 1584565 ps8 Te 4105 gx sttys30 Dt alder, aK 441, Descompuneti in fetori expresiile wmatoare utilizing, eventuul ident iibe Was Bla-hab (x ay(r~b) sau x2 at bx tab aly ale) i a) P6048: by F109 8205+ 96; z 88x95 tax +24; Ds -Tseids = @raisee$6; hy 82230460 Be 7x60 3 42, Descompunet in factor , 3 a) H5K+6; by P4710; ot aBrelSs E dS 4TH; M498 420, DSH: = ge aliee3e; Wa elleelO: DP AISe 486 : or los PERIANU - Dumitne SAvuLESCU ea FIANU + 43 aa, 4s, 46. a7. 48, 4s, 50. 51, 82 Dessepmmjuinele i facto abe Vee2d eeraes een oF 9548 DP Mao: Poi. 40 Wy -2by 36 iy any 75 Deseomponeti th fector: ay dens BFS @ Settee 12s ee eH l0 De 6-27 “355 Wy 8x15; pS -y-82 escempaineti i factor a) Stature ov sla Ser 8, eS a a$)-Sa Descompunet in procs de factor a) (2N° 43ND +34): 2) GU ~6eVSe 64 3)42 Ax Bap t y= 2ape ye F012 a (UL-3e4 Te ars 9) 41 ef be =D e3y SMe 42 Sra tse2L. Rezolvare. q) Obsurvim ef suma 2x" -+33 se gisele yin a dou arated Siodim. pent sinpisizeaeuleululs, 2x” 4 3¢-=e gave AV ONAN steed aclaed)e dea Hyde eeey as ged BY xmas aC -rlbs JF) bv DE +A aI av +342. Serieti expresile ca diferente de patrme si apor deseompuinei in Factors, ya +h! Bi Aa Bs dot 4B Warsi ei dat aa’eants potas ents whats" mn ols 6% aah Rerolvare ya tb! aoe +B 420 of a) (Bab = et ab» Buber an! = Sab]: |. Ardlali ed, decd whee Rl o+b4e=2anuncis 6? +e +2hee 324-1420 SaaVia #3r42)4b23-4 este divizibil eu 24, Ardila et mumarul ovicare ar fix © Za week Detesmsinati numerele naturale» penta cure numirul P= a8 +4 este pri Determinati cea mai mic’ valoare a expresiek a) of 4981 unde 3 eB Bu eeeib ge ve Tomde ve Deteiminati sal re mim a Hiecdeeia dintre expresile ) E298 ~65-3,unde ve ity OF FL. dx -243r+4), unde 16 Q: ©) Glxy=(2v-1)2e=5) unde ve By A His)28° bs +8. ude we 53, Se conser un tung ou ltule a le. Ara c ay Dacd wh! set + 2c°A* . atune) triunghiul este dreptunghie: b) Dact =o" #e° = ah +e =ea, stunci trivnghiul este echilateral:: o) Dack ab +bo4 a= ate+h'a + 2b, atune’ triunghiul este igoscel: a Numersle Va. SB si Ne pot lunge etre unl reg 54, Deterniinaié nymerele reste notate prin [ere ea in madelul da Bh PLP 2A Hae EL Gard Dn $2023) bal ex? bd Davee Pays Po dee gy eis +174 1) 3657 4499" —D4y41 = 12-1) - overs Rezolvare. 1) Flies se mii seri (a S94 (20 41 (n= 2F 1 Obtinem: = 82 22s mea. Probleme de sapte stele 55. Demonstati ed, penicu orice mumere seale.x.£, au loc inegalitile: Yar ary: Sayed: op Paher-9 De i oerty 56, Ardati cf pentrc arive eumere reales.» wweact adurt > 0, ou oe inewalitgi 9 ceeveal wxtter: 57. a) Fle «Sir doud mnere reale positive, Atay eae dey dy + 8) Art cd pnt ove 1 © ae foe mepaitatea - tt Bai BNE eidrtemle | dail 58, Langimil ntrior nu rung uot cs Arta ead afalsihact)=(ul eos) tune! céanghie este diepunghie 8 | MATEMATICA Clas | IWircee FIANU + Marius PERIANU + Dumitru SAVULESCU. TESTE DE EVALUARE Testul 1 veatgt ef tip) w Cokaby #(2) op) 4s 4041 91 FO) unde ve by Veriticayi dacs numsrut £(2)— F(2)+1 este patrat perfect, (1p) 6) Arieayich £(x}20, oricare ar fl eR (1p) d) Aritati ed £(x}=[FU)]’ :4 oricare ar fi ve (1p) e) Aralahi cd. oricare ar we Zen} = Bn) +[ Fn] este number natural impar divizibil eu 5. 2. Seconsidersexpresia E(1)= x(x=1)(«' -4-2)+1 ,unde xe Ns top) «) arses £(-1}~2(3) (ap) 1) Descompinet tn factor expresia x? =x~2 (ip) o Aritaics ora arf ef, numirul Eia)~1 s0 vide ou 24 (ip) dh Aritati ct, oricare ar fi m= Z, numérul E(n) este pitrat perfec, NOTA. Timp de lueru 56 minute, Se acond | punet din oficin Testul 2 1, Se considera expresiile: E(x) fi 1 trp) a avares 6(5}=F($) (ap) 8) Araati oe suretrul E¢4) 4) este pits periet {tp} ey Desoorpuneti in factor expresia FU) (1p) d) Descompaneti fn factor’ oxpresin £(6)— FU) (1p) &) Determinati ne2 pentru care Bon) = Fin)=9. 2. Se considers expresia Sls) = (2x +5)? “16, unde x= ® (1p) a) Deterivintivalearea minind a expresies Eta) pentru x © (ip) 6) Determinai voloacen minimns ¢ expresict E(e) pentru xe (1p) 6) Arttat 8, orieare ar fi n= ,E(n) este suds natucal compu. (1p) d) Aritaji ch exist oi €2 cu proprictatea & [2¢n)] este mumar prim NOTA, Timp de lucru $0 minute, Se scordi | punct din offeiw Fisa pentru portofoliul individual Numele si prenumete: lasa a Vilka —— Calcule cunnumere reale reprezentate prin litere, Formule de calcul prescurtat. Descompuneriin factori Tema nF 13: (1,5p)1. Completati pe fia spatille punctate cu raspunsul corect, 4) Rezslolealouluui (x=3)P este 1p Prin restsngeren formulei. 4a’ —(2ab+ 96" se abyine ej Rezltatal caloulualui 9x" y>(—Bay) +32 este (1,5p) 2. Pentru fiecare dintre enunturile urmStoare, dacd enuntul este adevarat, incercuit! litera A. in caz contray incercuiti litera F. ap Belx +4) = 3a" 412 AF by x 4d 42 0, orieare arti ce AF GQ Xumirol a= 2 x sedivide cu 6oricazarfireN. AF " ap) 3.Incercuitiraspunsul corect ta flecare dintre urmatoarele exert Dintre cele patru variante de raspuns, scrise la fiecare cerinta, doaruna este corecta. 2) Sind cb x= =H, vloatea exoresiel {e—yjt-+ 992 este egal A. 8. Git Be by aloacaexpresel cry pom Cate i pT er ee EEE Ad Bt 2007 : co) (2ayf3) (445) ete egal eu: Aba BTS CONT OH = 5 sisting 2-4. ature eps sy esto sgon é Ade Blas cL z & = 71 1 SAVULESCU los PERIANY = Dur 2 2 La problemala 4 15 scriefi pe figa de evaluare rezolvarile complete, pl 4. Fe numerete aya lati (2p) 5. Plocspresin Ex) = (24)? +207] 41 unde re a) Axitagicd EC) = [x 2(+6) ,pentry erie eB. by Caleulagi GI-1).—¢) Artiati cb E(x) +1620. pentru orice VER NOTA, Tip de hier SO minute. Se aeard | pune! dn afici Tema ?.i% Rapoarte de numere reale reprezentate prin litere. Amplificarea. Simplificarea Lo sapoe in care timenti sunt exprest slgshvice, se nimeste fractiealgetried sau exprevie alpedried rationaté © factie-algebrich nul ace valogrex de(initd pentru acele valovi ale varishilelor ware anulex2a mumiarul Exemplu. uci £1 s+ $i Gl 6. en Gis}=0, adies v2 tunel Fead= + ssle 0 fiaofie aluebricd, Spunem cd feaetia J este Gi) 3-6 defini pontra e421 od Pemmu eR {2}. mum) F(a) = AEE se mneste valoarea fracied Fin pnnctil a, De exempls FE) Amplificarea, Prin amplificarea tractioi aigebrice ©! a + Gad, co #2) Ex) FEL) expreia (33 # Os obine facia esta Fs) fine Fach Ge) Guy EW) . r, FO)-FUX) ny Fis Fey" Fin Give Gos expresiu E(s) #0 se obtine tenet Observati 1. Suma gi prodiusul « dowd seri algebview sunt fiat algebriee £09) 2. Dac Fisk = PS cw Glad 2 Osi Els) #0, atures expresia (F(a) = este Irate algebriea, * 1. Detentinay) valorile vaviabilet © pentru eare Hinctia dana mu este derinite MATEMATICA Claso a SAVULESCU | Mircea FIANU + Marius PERIANU - Burnt l 2. Se considers facia E(x) = a wuinde 9 eR. Caleulay preys mat oy FQ) "5 . ws os 4. Amplificuti cu x--2 fracjiile (#2) 2 xa? 2x a oe 3 ye = ? xo? OFT . 5. Seva rate ezltate in ura simpliiete (1¢> 0) 2" a ay wage? Ors w ; Yrs 5 9 att 35 WE | ee ares 6. Sinplifayiou 43 fefile, ee R\G o Hx+3) | yp CANO) o (+3)! (es 7th xe Baeer 3) Gar) ieee 7, Simplificai fregjile (x>0, y> 0.2 > 0) a ae 2aety? 2 at, yg wg M ” ae Miah) Ose He we 8, Menjionati condlitite in care fractile dave sunt definite gi sinmplificat-te ; YEN, goats) (era ip 23! F-deeD, Iaee2y 9. Mentions condiitle in care fraciite date sunt definite gi simpliticay-fer (3X43) ea » es, oe bn 48 ore A 4x8 420425 ) rae I x+3x" 43041 3x 43e-1 yee y epee a(t e254) 1 Sara ay 160 aoe sere fc variables x pentru core fiaefin dats nw este cleft D 40, Detecminsfi vaboril ae = rd ae “Sah aes5 Bett! to yp ? ame ODED » bent yy TNR Bls 3) m) Feaat [en 4)ie S006) Catouiat: 11, Se sos pone (0 f. By gacovren, e(t}re(2)s 9e(-B)er weconns oid): a(S) 112, Memionati confit de existents gi aduceti fragile fa oeelagi nemitor: Fe gy 723 ° xv) ‘ Le ty, 12g mH » 30% Bae wo « fractile date Sut detinite gi sinmpiticai-ly 25x" 20x ~15¢ 65s 14, Mectionayi conde 1 5 oe Sg Mae > Ise 38455 1 Be 7 lax* ~ Sx 2-3 @x-3) oy te 2 gy MS e2he ° Sa(2x—3)" i Ge 9x" 9 Wa 15, 415, Mentone eondtile ta eae Factie date sunt definite si simplitica-le sa Se+6 ya 9 ee were oaeele Peter? s MATEMATICA Clasa a + | Mircea FIANU = Marius PERIANU - Dursteu SAVULESCU 16, 17. Pate ‘Memtionari condiite fn cave fractile date sunt detinite gi simpliicay-te 2 Rees jy aD 2750 13s cnt yt ny fa pe FTE ‘joa comelivite in care rstile date sunt deine gi simplifctile yp GteTE a5 gy Ged +2) ir 2h Toy Wa5P led)? 9 Be o (404 BP ~(20-7 waved Ga-3F aay Menpional eoniile tn care srchite date sunt definite gi simpliicaite Bae +2be+ 2ab- Bae stay —tOrs +35 oy APE i = at inwermin—bymey Taye e210 wkiek 21, Meni coi eae file date sunt dente spline: vricetees yrds Waa Soa) OS ed 22, Menjou eile y ears fale ene suns spite Sat =Sab—Sa, gy MADE +a die| ? Ral Ra Rah Bar oy TA Mos oy 4 3t0 4o) t1 (2e13) 7499 (avapro ‘ eH Det 4? 6 273, Meotiona condi cate eile dae sunt deine simpli icatees gh Sees} St dee S)6 ete sie Bet} Geyer (SHRI eT so (x -Peet ifs! 7043) +4 Haga) (oopieaeaed “fanaa forties [ef ac-9}fatex-is}eo Oa dreepes Poa e ne npas Probleme de sapte stele (ona +8 r= 4)45 (55) STs) oz 4) uljines Ds unerelon x © peat eae expresia este dont 4) Ciementee multinii 4={veB| Elsr= 2}: 24. Fi expresio Ef) Determinay 6) Blementete mull B= [: « wade! +8146 25. Se considera riportul F 7 1 Flay-3 Arita ed >O,oricarearfi v= R-{~If 26.Se considers facta algebrica Us unde xe R fst, 5} ° loex 1 Dacminsemintle walt {re a| oes MATEMATICA Clase 0 Mircea FIANU + Matius PERIANU + Guriteu SAVULESCU, | Tema 1.47 Operatii cu rapoarte de numere reale reprezentate prin litere 1.11.1. Adunavea si sedderea 1. Bea 9s 2. Efecwuatis 78 OTT ae had ace3 2 Seed San a o xe=3) sl 30 ” eet yal ine? “1 ’ wart x03) Li S3x xl wal wa Bes! Det} B+" r 4 Bfscunyi: | 17-28" PB (e ard} y= * wa 6. tects ork, 0 Te Be iy r-5 Gepard @ 1248) Va se 1)[Be—a) eos Beas ead) 1x3, HFT ot Baa & KK res) » Salsa Bee ard f | MATEMATICA Closa a & 3 a i 10, un 15-6 Fiv(reat” ia(re 8) ) os oy 2. keeinat eel as a See 2-5. seed) o Freewa Efeewat oe en o By a39 wo aon 3 Maes ox GL » o ay wu 1 Ets Varas7 5-45 2 Pax) Steet) avian ]) sa 12, Electway i = Rear bre) acta bre) aia be 6 a 13, Electuatl xe o 5 o aa ela2s sxe 3 (et ; Sox dex 1d 1A, Se consider expresia El) Be * &) Ste Soy 9 4 Determiagi numero rete pent eare E+) nu exist by Arttagi ch E(x} = 2 ©) Deterinati nutnerele ineegl x penttu care Els) eZ 1, pectcu oricare ae RY ea. aarR) a ave V5. a) Atti ed by Pentru a, KEN" calewlati sumole: tot —t ss owl) (ar iKaed) tw t3yerd} (4201 a+2012, 5S [ruremnercrsn aot YT 1.11.2, inmultirea, impartirea, vidicarea la putere. Expresii cu toate operatiile \ GalF PT ee 4 Periny ce valor wale sles expresia nu ae volar defini (nu ae sens)? ay 4? ie expresia Ey by arnt en EE) 1, Brett .) Recolvatiecuatin: 7. Ey) = 5 wh Determinati volorle pe pentrucare 4.y-+1)-F(y) este aumae inreg, Soviet eu ajutaral itervalelor mukimea A= {y14} x 42 Fie eapresia. £0) oy a 4) Determinali ve R pentre care Bx) nu ace valosren definith by Adhicettexpeesia EC) Ts forma cea wat simp ) Rezolvagi esvatia: Ex) =3 d)Catewlati peodusul P= E(2)-€(3)-£(4)-.. £2011). ar-5 (ort) oy enna $= Fa) B44) + B66 E02). Cala Mircea FIANU + Rarias PERIANU «Ourvitru SAVULESCU, & |raenancz cn ava 4 6 oh (Ss ay AOE 6. Coles Gel peels SOL 2 Gee hore) sto red GDA Der FFs Boar 5 > Calenlat oa) 11. Efeotuat is 4 os ‘ f 24 5 oft); 2 4ax¥ | 2 8. Efevtuayi: OD Te (sy l | sis? 426) _mvr6 2 ay3 18 ll 1 l a mes Ix—5)" ” “25 Gt dead tw H6r+1l sl ady ed 66-305 z oli 0 Sa me 28 Maye 3 : 2, Fete a ® Tey 1 se sagan—3, 200" wo | d Heard [ay 035 0a aes 9. Cae yp m0) Sant 819 py 2 atte S53) a 9 Be2o=78) Bx (2a 70} 28 °Dr-3 Galeean ree ve 43, Caleulat ols: ince EIANY + aris PERIANU + Ba MATEMATICA Clasa¥il-8 —1 Mircea FIANU + Marius PERIANU + Dumitru SAVULESCU 15. Descompuneti simplificati gi efectua inamfrile wird Sty ” 2 Ay -25) Afx~3) o Sel ye 46 +e +2ab+2oct Zhe Bx? +15: 8 49x" = 25 jasby = 16, Bietuni: o Lise sos yy Soha T Se Dv-8 95-4 dre ae Pea @ 3a 3). Sat =a” eld tdo oe? OF OO Sa Oras ae gy Att Gate ze (3 15-385, Slt 2a ey Ory Tab 92 100—20x+1" x/-64 10- 7 ~aae=8) ade Toe AB we 17, Bieenat ale 2a\" {2a} 3x9 Pf 3x9 J" (3-3 ABE GS Gal Ges fmt (aay (sey (2x) sy] 34 y (25) 1) ) Pee oP) GY) [S)]s x49 24S 4x? 200428 Ten 2 » Beciv'e 25-1 Vio Baty) Bendy! VM Br ros Pads? e3e4 erry 8412 =25 Sut Fes10 x36 2e410” 36 is Qe waar) 4 ass 1a, Erectus 2 wot Ten roxe175 10 lives 22) 24S” ay! (SS) as ee 20. Se considera expresia aoe(ey wal ea ‘a) Peniru ce valori reale ale luix expresis data nu are valoszeu definiet? ') Aduceti expresia Je forma cea mai sip. o) Rezolvaii eeualia FCs) =13 self.) eenpicsia Py= 7-2 wl 21 Feeapesia Eo)= 7-22 HL ‘a Poatra ce valoeireale ale lux exprosia fu are valoarea det nit? 1) Ardtati cé expresia are valoare constanta (nu cepinde de x) MATEMATICA Claca aka 4 SAVULESCU KANU + Marius PERIANU + 2 Vieesprests Bel = 23. 24 26. Dr, 4 ‘4 Pentru ee valor ale ui vexpresia nu ate sons? hy Aduceti expres fy a for ta simp Sasifol inet tr) The mummy ineres nal ©) Detesning by Daversa o incat £(a) sh tle numr introg dives cu 3 te facia (oyy] 2G EYE FR HAO 25-09 Tae 28g a) Sirmpltiost teach DE Cateulatt F2s=1) oa 4; Determingyivalorie tategi sle nt a pentmu cate Fta;~2a+3) 22 sult Plus 243) pentra «eH (451) Fic expresia (2)=a°-S0? 44 ey Descompuneli (9) yi acta et, pent vice w=, Efe) se divide eu 120. (DEW) ay Nexiieay Sa} 43062) andewce L yice Hea yen ay OF 238) B82) dens sh 242: «) Reobvas cunts — Aaa} A Cab sua $= FO)= FU) +o POON), unde Ps) 2 AEE Gy c suse bat dot) a) ea G3 Pd da) Seed FE) Temp 4) Penton ec valor reale ale ui-x expresia & nu are valoarea denice? Jy Sduecti expresia. E(x} ta fouma cea ai simpli Deternalx & Z pony eave E(x) eZ, 2 Seri uinen = [22 Bf|260- 1-693] <41} fosin interval 2 Reed 4) Allatis=.R pentru care expesia BE} are sens, Se considera expresia: (4) =3-4 by Acaiai e& Hex) € Bd], oricare ar fl xe: e) Caleulasi pastes ‘nireaga @ numaraloi ECs}, pentru xeR Bri 48r—6. FID sale ale luis expresta & nu ate valoarea definita? Fie expresia Bly Pentew ce valor 1) Sinnplifieati Ee) 28. 29, 30. 3 32. 33, 34, 6) Decernsina vatovle Drei ale ll v pentru cane: Ex} ia Saloni inten. recs ne(2 2) wep 4) Caleulayi suns valorilor lit 9 eR poatre eare expresia 1[a) nu ave gens. by Aratagi ppenten care exprosia 4s} are sans 8 42f€3)<9 ese peal perteel pena osiee nemar inereg © 9 Bxisi valor ingegsle hx pent cae (x)-(2—5) @@? - Ficexpresia Exx)alx-100]4¥—99|+.+|y—I sie Ife 99/4] 04100). a) Asatafi cd E(101)= AL-10)) gi caleulai seeastt valoaze, ) Stabilti daca £10) este mumar par, 6) Auttatie& decd 22102, atunet Bia) = € + _ way 10 4 Pentru ce valor ale lui expresia ave valoatea definita? bj Arig ct Eine (5,6), vee 6) Determinati paren tonreaga din EG). 1d) Allah x6 pentru care postey inteesgt a lui B(x) este 6 Fie expresia E(}= 5+ eoya| (28) 21428428) 5,9 Foexie Fu=|(248) 4142842] 82 4 Pent valerie expres na ae valnaren defn? 5 aauses expen fore ca i so «) Determinafivaiorile lui € % pentru cane Ba} este cumar tnteg: 1-5 4d) Resalvayi ceustia BLs=4 Facey Is forma eve mai simp Expresia (+) verified egaltaten (y! -¥)E(x)+13 -4+ | @ Caleulati Ea) pentrn ce RY HLGI si aducet by Caleslai sna $= £3} + BA) B(S)= FU) ©) Gasiyi numerele € M* pontry cere partea Honrewwa lui Bts)este menu 18x" =35-10 Fie expresia 6(4) = ° a Ox F12e +4 4) Caleula uma $= (=I) +£{0] +E (1); 1b) Simplilicali 613; ©) Exists valoriiniregi ale dai pentro care £(+)ia valoni tntregi Fie expresia E(<) =o? —1s" +9 1 Descompuneli £(x)5iaritati c&. dacd x este un aumtke inreg, multi de 3, a avila -1 x | Mircea FIANU + Marius PERIANU - Quiteu SAVULESCU 35. 36. 38, 40, avunel £4) ese vith ou 27 4 Verificat dat (0° ~)E¢2) se divide eu $80, pons ve “ DEO ps 9 oy Nestea ed BEDE (4 29} 44003) & Cateulayi suma $= Fs FQ} 1.2 FANII unde #(s) «LAKE Buy 2r#] re] Set ese toe) Fieexpresia Fi) unde seasfat.o].aaunes #03 Fieexpresia E(x] ie pea ay Cateutat £65) 8) Acitai cd (+6) —x-6 = (9 +5r+6}e+7) 6) Descompunesexpresia E((#-+1)))+ (4°) 12 unde & este numar rate 1 meth a) Abus et: 2 a i +b) Careeste al cincilea termen al somrei + “Tata a7 ©) Acatsji sua primilor cine’ termen! ai sume este wn nude subunitar. wats! 39-6 vod a) Aciaji el B(x} se simplifick prin x42 2b) Calculate (1) £(0)-£ (0): 6) Fig me N, #23. Veridicay dace (0 —2)E() + (a +2) este pateat perfect Fic expresia E(s) stnde re RA{-2,2) Fig expresia run = Zt E48 | pres FO TES a6 4 Arita ch FG) =14 > pentivarice x 0, orteare arti ye R by Affati xy ¢R pentru care £O.y)=7 46. 4) Adueeti le Toxmna mai simp surna alyetrica Se4 (x HEE) stacy (ena REDE 2 by Determinasi valoares mnisims x expresio: Ela, yt) = Sx £109" +22" — ayn 6) _givalorile rele ale lui x,» si= penlre care se eelizess yg SE SOAR 7 4ve]2 +1012 seest minion aT. Fis expresia E(0) a 4) Sinpitig fneua B=5O= AE | MATEMATICA Clase 2 Vi sAvuLEscU ‘Mircea FIANU - Marius PERIANY + 49, 50. 53 1 Acta e& Blo este ptt perk €) Deteeminagt eel mal mie nue penta orice ne Nn 28. F natural penn cave F(x) se divide la (08, + FEI) sande Pes) tt Wea) ef Calculayi sunny $= P+ (Oe Posey es 2 youl nent ye et as Cale sum yy 1 Calaiaiesa ae noe pi inode ©) Arita cd pentru orice yy'€ R* are foe rlatia Probleme de sapte stele A S_.2xs3 Se ddexpresia Gt 3}, ty) \areS e316 =25) V6 bane 2s 11) Detorminal velorile luis pectcu cere expresia ma sre vatoarea definit By Adaceti expresia la Souma cea mai simple 9 Determina valorte intregi ale lwix pentru care £(x} este nunxie natal @ Rezolval ovata Etsy «8A Fearsome Qesect went &afeet| a * a4) Pai: MAM CH ELE, pantevorice we a Bixy= + Se vonsiderd aumerele reale pozitive a 8 5. Ardtagi ef 3 \ U poeta athse” ortbte atbste atbee ath bee oha” srcare ae ff] tosh? oa 40 Se consider numerele seate puaitive ag Acta ca os * BS erbP aap 3 Preset WF +P a Sdat yes), by Avaya freee + frieare ar fi inumerete peale poative x. 51 Se considera mamerele « =a P= unde vem {0} Sila c# mumercte a si sunt rationale, aehaie8 4 este numa rational ap) op) op) tp) itp) fp) op) ‘app op Now Op) op) Op) py up) 2.Se dau expresile Et 1.Se considers expresile: B(x) ’ E TESTE DE EVALUARE ‘| Testul 1 4, Se considert expresiile: Fs sande se BE a) Acted (2) = 6 \ iy Veriticag dock F(a) =6( 1}. ovcare ar eR! ©} Arata ca, ovicate ar fi x eR expresia Els}+ Fis) are aceeag! valoore, U aye! A Netti A, orcate ar eR deck E{b)> 4 arunci PES) <1 2) Dowrminadi numerele ee I penn care Ee) Fox} = 0 4 arsopies Elvi) =2V362 sy Verincay dacs (V2) Fi © At cid HERG. b>) otunsi BY e(0.), -A, Timp de lucra $0 minute, Se acon 1 panet din oficiu Testul 2 unde xe, sy Cae 5-5) by Sita ed FES) ©) Dedcetict Fede (el. oviee ar fh ve o) Arsiati es [E(mft +[ Poof =I, ortearear ff ve R, 2) Deducet ed |2(s3| <1 oreare ar ve MATEMATICA Clas a Vl fs PERIANU « Dumitru SAVULESCU {Mircea FIANU = Ma 2. Se considera expresia BO.) = 3 unde xe ara Ap) a) Cateulart (1) (1p) A) Simplifieayt expeesia FL) (pt) Determinag valoatea minima a expresiel E(x), unde eR (pl) Arduajied, oriewear fl me S, Bm)~m este numae natural par NOTA, Timp de Iueru 50 ssinuts. Se scords | punct din alici Testul 3 1, Se cunsidert expresile: E13} tunde ve Nt mad (1p) a) Csleulati £2}; U A. oricare ar tive BY ert en (Ip) 6) Catevleyi £0) + £2)+ £0)+...+ (10) (1p) bp Aratatics £Cx ma sn46 Fry ‘este oumar (tp) dh Aritayi ca, pentru orice eM, annul natursl divizibil eu 24 fp) od Acitayi ci (0? +5 +8}-£(n) $6, oricare arf ne NY sttattl 2.Se considers expresia £(x)=2 5 tt cree punde xe R ooo ca of) Cpl by Arita ck (e+? > oricace ar fl xe. (pl ef Simplifies’ expresia (09 (0p) ap Arstatiea EG) este numar natural impar, oricare ar ne N. NOTA, Tinap de lnere 50 minute. Se acerdf | punet din ofiem Fisd pentru portofoliul individual Numele sipreaumele: Clase Vila A4 Rapoarte de nurmere reale reprezentate prin ltere. Amplificarea st simplificarea rapoartelor. ‘Operatiicu rapoarte de numete reale reprezentate prin litere Femanr.i4. *41,5p) 7. completati pe fisa spatille punctate cu raspunsul corect : 2488 49 Pein simplificarea raportalus 4°55 se obyine AL qu este definitk daci x are vafoares 2b Praetia Penta + 20, rezulatul calealubos 252 + (0,5p) 2. Pentru fiecare dintee enunturite vrmétoare, daci enuntul este adevarat, incercuiti litera A. In caz contrarincercultilitera F. @) Pentra x51, valourea fractiel 244 ose h a F aS cerultatulcaleutului (2—] Ay este | a F ©) Fractia at ‘este ireductibila, penicu orice r= R A F (2p) 3 Incercuiti raspunsul corect Ia fiecare dintee urmétoarele exerci Dintre cele patru variante de raspuns, doar una este corect’ salar +24 a) Sinspliticdind rapoctut prin eS se obtine BI aS co pric ae si veatta.ysa-1 iret Anne’ 2 este Ae Ba Gast D1 MATEMATICA Cina Vl La problema urmatoare serieti pe tiga de evaluate tezolvarea completa, (26 os 2 ates a) =35 = unde opp ai txo6, onicare vez, 2p) soriare ee BAL 3 (plc) Ata vatorte nteepl ale li a pentru care Ete TTT NOTA. Timp de luer 50 minute. S¢ aeorda | punet din effet Tema 3.7.2 Problema cu caracter apticathc Rezolvarea problemelor cu ajutorul ecuabilor presupune pareurgeren tunngtuaretor tape: ¥ stabilirea necunoscutei(necunoscutelor) probleme. nokarea acesteia (acestora) > serierea datelor gi relatilor gin problem, objinanduse, astfel, modclul mate- mate (ecuatie sau sister) al problemei: 3. reaplarea ecuatiei sau sistonului obtinnts ‘ interpretarea solutieé si formulares ispunsuli [a problema. Exempie Problema: Un catator « parcurs, in prima 7i 1/4 din intregul drum si inca 30 ki. A doua zi a paccurs 2/5 din rest gi inc#! 10 km. Care este lungitnea rum, daca pentru a tea 21 -au mal cimnas 80 km? Rezolvarea aritmeticd, Folosim meando mersutvd ssvers combina’ cv matodta grafted, Reptezentém, priatrun segment, Sntregul drut: 204m 0 te 14+ 89=90 kan inseomnd 2 din primal res (drum rimas dup prima 2) 90:3 $= 150 km este primal rest: (150= 30): 34 = 240 ke este Iungimea Tnteegului deus, Rezolvarea algebri Nowiin cv lungimes intregutul drum is prmaatea pcre Leese, Armas a-(evau}-Ss-30 (imal wo 4 (34-20) 2 a sl Gste} toe oe este distema pateursi a dova zi iw H 3 *. Se obtine eouatia: Ly 34 2.5-2480-x cu solutia r =240, r i eae E28 i 4, Lngime Inte gnl drum este de 240 km. Problema: ini-an bloe sn: mines spartamente ou 2 eamere si apartanente ct 3 samere fecae, tn fetal sunt 25 de apartamente care au torte 69 camere, Cate apartament de Fccne fel sunt in bie? Rezolvare aitmetica.Foisim meta flse? ec, Presiputem in ble sin swumai aparatrente iit Spier. Aton 3-25 = 75a ral de camer Dilerena ital este 75 — 601 ~ 15 cameve. iar dilevenia arta este 3 ~ 2= | MATEMATICA Cla32 3Vil-a —1 -rcea FIANU + farius PERIANU = Oumitey SAVULESCU ceamerd, Agalar. sunt (5s = 15 apsrtamente cu dowd eamere si 25 - 15 = 10 _ugtaniengé eu ie earner in blacul respect: Rezolvare algebrica. Not ev murnfrul ypartamentelar eu ede 2 eames. 2, Regultd gh vor fi 25 3.Glniner eeuagia: 2e-+3{25—0} = 60 eu solupia w= 15 4, Sunt 15 aparamente cu chao 2 camere si 2 cmere figeate. Proba: 35+ I= 25 apartamente si 2-15-31 = 15 © 10 apartamemie cu ete 60 cameve in toca * 1. Ma hus w cumparst an ereson $i 0 cate pe vare a plauitsuma de 4.8 fel Silind 8 prejol chrjil este de trei ori mai mare deeat pretel ereivnului, detertinat prejul Hecaruia 2. Suma a dou aumeze este 44, iar cliferenja Toe 12. Core sunt wumerele? 43, La unica au sosit pe rind wei nepoti. Primului ia dat 1/4 din numa de ere dintr-an eas. la al doiles, 1 div nuuvdral serelar eAmase in cos. iar foal tuvilea 2/3 din noul rest si astfe! am rma in coy 12 mece. Cate mere « primi Fiecare nepot? 4, Perunetrat unui drepunghl este de 26 em, Daeg se mceste linea vu? em si se micgorea2i lungimes cu 3 cm, acia dreplunghiutal nu se scbimba. Caleutat aria dropeunghiul 5, Suma a dous numere este 65, iar diferenfe dint tiplul primulu si jometatea celui de-al doilea este 48, Determinayi numeseie. 6. Determinsti numaral despre care gtim 4 Tnmultinduel cu 3 se objine aeclagi rezsltat ea alu end il Adenine eu 486, mee gi kB} = 6x ial este isoscel, 7. in wiunghiul ABC se cuno Determinat valorileIui.y penteu care tung 8. Un automobilis a parowrs 270 km In 5 ore mergind, fi 50 kenth i locabihti si 60 Kru fa afar loealitagtor. © {in afira localitiitor? 98. Cu o eamitaie de moccovi pos Jk helnigh 11 fepur timp de 16 vile. Pentey cite ile ar fl necesari ncesagi eantitate ca 84 hraneasea 8 iepuri? 10, La un magazin s-a produs 0 redueere de profuri cu 15%. © rochie costa dupa reghicore $1 lei, Ce pre] aven rochia inainle de resucere? 11, Radu eiteste 0 care ase in prima 2i 1/4 din outrul de pogini at ede to & dows zi 146 din rest a treia zi 3/5 din noul res, iar pentru a potra 2i fi rimin de citi 42 pagini, Cite pagini are eartea pe care o citeste Rad? 12, Intrun bloc sunt apariarsente cu dows camese gi upartamente cu tei eamere. Swind c8 in (otal sunt $0 de aparimente gi 117 camere, determinayl cite aapartamients cu doua camere sunt in boe edie, eu viteza de i kilometri a paveurs 43. Cat a suck din media avitmetieg a aumerelor 46, 23 si 51 este nuntirul 6? 44. Davs fnuro sali de clas se agaza céte 3 elevi inut-o haw’, sdimiin 3 bane! goale, iar daed se agoed ete 2 elevi inte-o Dane Rinu 3 ckevi Th prewave ‘lovi gi ete bine’ sunt tn elisa? 45, Tn chast a Villes # unei coli numdnul baistidor este eu 7 mai mare devi hhumitral fetelor. Dac din clagé at pleca 3 Fete stat veni 3 bie! ates ar fi de loud oi msi emul biog) decat ete. CHG levi sunt in clus? 16. Teel elevi aw inpromnd 3200 fei, Dup ce prinml a eheleuie 1/3 din paca sa. al doiiea 174 sin partea $2, lar al ercilea 3/5 din parlea se, ou rimas roti eu sume egale de bai a) Ce sum a avutla Taeeput ficears 6) Ce sum a cheltuitfieeure? 2 17. Livan concurs de tr amar ermeioreeprezinid din nema eooeuren- lor. Stiind ef sume diatre numarol concurenjlor yi numdral armelor este 156. ine fine concen a foe rer trp nue de sane cere ou fost prema 18. Un potion costa 400 (ei, Bupa. douti reducer de pret paltonul a Fast vant eu 306 Tei. Prima reducere a fast de 13%, 14} Cat a costa patonul dupa prima veducere’? 2} Ce procenl a fost a daw reducers? 19, La un coneurs Andrei, pentru 8 rBspucsuri s primit 38 puncte. Caleulati eine rispunsin au lost corectegtiind c& pentru flecaze rispuns eoreet se wcorda cite 16 puncte, iar pentru fiecare raspuns greyc se pierd + puncte 20. Adan ile cin’ ori pitratu] una sumde natural cu seel name ondrit de 7 Fi se objine soma 24, Care este numeral? 21. Pentru 2 ky moreovi gi 3 ky rosii s-a plttit suma de 15.5 lei far pentru | ky imoreovi gi 4 ky rogii sa platit soma de 16,5 lei. Cat east 1 Ky de moveov yi cat cost Tk de ogi? 22. 43 muncitori pot termina @ luerare th 18 vile. Sting ct dup 4 zile 6 muncitori au plecat pe alt santier, aflafi i cite zile este terminath luersren de eatre smueitorii rama. 23. Se gtie ck dou ereicane gi douit radiore east 7 lei; wei radiere gi evi pisuri costh 18 tei, iat vinci creioane gi cinci pisuri costh 32,5 lei. Cat cost un Creioe, doud radiece i trei pixuri Ia un Toe? 24. Trei copii au Hecare edte o sma de bani, Primul gal doilea ax tmpreuna 393 lei, al doilea gi al treles ay impzeund 395 lel, iar prime! 1 al ureiles au fnapreunh 326 lei. Cagi le are Fiecare? 25. Determinati doud numere Introui a elsor sum este 28, iar diferenqa pitratelor lor este de £68, 26. Diferents dint triphul paerasulul unul nun catural gi aeel munity mart de 20 cori este egal cu 7, Deteruainayi sumarul MATEMATICA Clasa a Mircea FIANU + Marius PERIANU + Cunites SAVULESCU 10, Fle « Tema 4.73 1@ pentr.t Derformanta scolara si alimninde Cale: fo 5 eed, a 2007 eu ye? 52007 = 2008 duet ae Fie C= no-tma! ea cnt + 2mm borko Antioch £52, oscars or fh rurmerete naturale a gi Caleulat peocusul abe stiind c& Pr aatorstofons Ardtal c8, pentru orice 9 = Ml oumarul Fie expresia: £(s)= Vimar 8a? Jae Se T .Caleslash (20064 fd — V7 a3 Finfri)a7 ext rational a Coleui «: by Cateulati 9 = (1404 2v3)™ oj Avwaticn ({7-aNa) +(Jrea)" 22, osleae arti vez CES — VS4V9-W5 gi v= \vi-i-yli- avi Acttayica LEQ 4) Caleta poral “unde: N= fo= WENT TAB i Vario + VB dBc +2) 1B) Ardap ch expres aan pumare intreg m0 6 (ty See Bf) SPH. Caley ers) ante Asta t Jn oricarese ne RT 3 Fie 119 (vise +68) \ 1 \ BE DRE elratatnntide by Artal ck dock EN, aunci VER VES +e fmGrel) 0 1 Afla) vulowea real e nomiulul B= ee atte sintea(o-t] ve 33. Fie ab eR asthe facét o? 4° =2. All ceo mal mate valoare « produsulu a-b(a+0) 3a, Fle mre? ta >. Anat enum ar dn nu este pr, 35, Numercle a,b i esuni pozttveyi aeb+e =! L 1 24,3 arb bee ote Demonstrati oi 136, Fie xy giz numere reale astfel neat 9° —6 = 7. Wy 2 =| si leo = 88 Demonstrefict #3, p=5, 257) 37. Dewrmingi abe © Ro stiind e@ au toe simulta retaile st L ot poe e ty on 4 4 28. Fie a, © R astfel incit a? 4D! 3a She = 0. Aratati ca A cath +s 6 7 Into elasd sunt 20-de elevi, De Sf. Valentin, feeare fad ufesd feed bata tes flori g fisetrel fee @ floare, iar fiecace bRia, oer ete tet sles ieee Fete gi wie 0 lease fiewtrui bait ‘Avital cA nurirul masim de flor ofeite este 780. 1) Cate ete ae teu 9 fe in clas, asl nut st Be wfette eau 7801 de Mer 40, a) Arktayi ca 4g by Aftayi numerele x, 75 2 € care verifies simultan ewalitaile wv ys =1 si 2 a aye yeece, orivare ar fi numerele realex. 91-5 Sy ay haee eye 41. @) Dack oh, €R Gi at b+ e 20, athigied al 4b" +e' = Babe 2 = Ay Numerete aie pezitive gb verifies eetayas “ts Jah = 2 z ate CAPITOLUL Ardtali cl =o Fie o gi 6 dowd numere rationale diferie, @ 20. Considerdm mutimea ts nate} oundle =a. %y 28 giv, } Numiti tee plancle dterite dcterminate de cate tre dinine cele cinei pune, toate planele dferite at MATEMATICA Clas a Mircea FIANU + Marlus PERIANU » Dumitre SAVULESCU . 1 16 20. a. 25, 26. Se vensidoctpunctele 4 2, € © $i. Beterinsti mart de plane dite dseninate decree cole ene! pate, Aralai toa ale posi Jn fig store puncte 4. yC sme state fa ple far panel BY este ester plana Completa spe punctate eu spurl eorect. o cataryicany 1 (ansciee op MBE = a a tena " 9 OBO P(BsCine ; BCI hy ait a Stabilii ounare? wining mumarul maxim do dept pein care pat (Fumie, dowk fte dou, 6 puncte distinets, 1. Demonstrate, dae Tn spafiv tei dropte neconcurente teste tre so intersectewz lava ette dows, atunei ele sunt coplanare Se dau punctele necoplanaie 4. 8, C. D. W) Scriei tre drepte deteminiate de vice dul din aceste puncte, 4) Series. wei diepte deverinate de te doud din aceste pune care contin punctul 8 ©) Sericti ei plane a caroc imerseetie este punctul C. t Serieti rei drepte enplanae toate tei ©) Seciet rei crepie eoplanare dud eae dow, da necoplanare tate Wei Bout plane listinete 1 iP se intersecieaz’ dps dreapla AB. Punewul M este sitsst af8¢ in 8, cil si in B. Ardinjie€ punctul Af este situat pe Geeapta AB. Se considers muljimea de puncte }4,8,C; Df ‘#) Dotesnsinati nunsiral de submulgimi dc ofte trot puncte ale multi dares 25) Duet dows diniresubsmalyimile de eri elemente sunt formate cu puncte ccoliniare. arta cf poneicle 4, 2, Csi D sunt coliniare Se considers multimea de ponete 14.8,C,D.8 4) Arftagi ci, daet oricare toi panete din evle cin cele cinet punete sunt core by Dues punnat dout surpmultimi de ete tei puncte in eee cined sunt formate ex ponete colinire, rozals cd cele eine’ punete sunt eoliniare? Justitica “Teei vrabii clugulese wrouage de peo supratara plana, Lan moment dnt toate sei boast in dieectsigiferite, Dupa eft timp se vor afla intra aveleg plan? sunt eotiniare, ananei ware - Se consider multimea de pute {4,8,C,0, 8 a) Daci osicare submultine de efte patre puncte din mmulfimes dat este Format cu puncte coplanare, ariatt ef toate cele eizc? puncte sunt coplanare: 6) Duct nomst dowd submulgimt de edce patra puncte din vele cinch ‘cu puncte coplanar, rezulth ef cele cinei punete sunt eofiniare? Justieut Explicatio constiuejie a dout crepte necoplecare. Se dou punclele 4, 8 C, D astiel Ineit [D4] Demonstaj ec [4B] #[8C] tn urmdcoarele sini 9) 4,8. C§i Dun eoplansees HA, B, C9 D sunt necoplanare [BC] si 8. Determinafi valorite (ui x tm eal cind punewele fi. mu dover an pl. 29. Teapens| ABCD are latura neparsiela 42) ineloss in planal ‘ntersecteaza planul ain pune! f 44) Aritaj e& punetele £4, © sunt cotininre: { Dack AD =8em, AB= 24em si CE [WC] in lecsee din urmtoarcle cazusis » DelAB): iy) Ae (DE) cm si a= be spe Dreipia CF caleulali ngimea segmental Probleme de sapte stele 30, Se daw puncicle nevoplonare 4, B,C. Psi M & [AB] Ne [CD], Determinats Inerseetia planclor (NAB) si MPC), 34, Punctul ¥ est exterior planulti pateatului ACD in care A sn vedere aceastt confiyucae a) Dati tei exemple de tei penete coliniares by Daji tsi exemple de tei puncte necoliniare: 1) Stabilti daca paneiele f, £, 0, F sunt eoptanar sepinentelor [48] respectiv [CP]. 7 4] SS ‘9 c/ Repcererigi dou desfiyurtci difrite ale unui parlelipped! drepronghie ABCDA''CDY ex dimonsioile AB=2cm, AD-=3em98 Ad ~ 40m, 0 pris hensyomslt ave bazele hexageane rogulale gee ltorale peste O muchiea pine! are langimen egal 10 en. 4) Repcerenia prin dese pisna cat 2) Calcot suoa lungimilor itor muchilr prism 6 Cale suma tts rior flr lateral. Cale ain baz’ priate sem gi Wid em, BC’ =6V5 om | at RK 21. oul pivamie pglatere au bard comuna. fe vf simale de'@ pare se aia plalu Boze, bic fy Nunel de pus sle poled obit » fy Nae ee ee ale plied bt US snout feb sr gua iat san ES | nO intersesiadiagonsslor AC" HO ee. deh pon Otrex gi lagonalle C4" yi OP. ; By Dach_aB = 12 cm ealcelai volo picomidel 4 GABCD (obseevind din ete antel de pramide Se 3 feompune cuba). 23. 0 baz a uci prisme are nmi Expat i funeie de lajcuredeu de vtur ole prism 8) mae ttl de ee ae prise c) munarul total de smc ale imei { Notion cu y nursitul de vécturi ake prismei, cu ns nant tet ee visi eu f rumiral total ds fee. Verifeati dach are foe egalitutea f+ = m+ 2 24, Stabilifi valoarea de auevar s propozifilor 1a) Nuvo total de cd ale ue prismie se divide eu 3, 1) Dow prisme cu acslasi nude de varCuri au acetasi numa de fete Dou piramide ou acelasi aumir de feje au acelngi numar de muehl ‘ Doxa poliedre eu azelasi nurnir de fete aa aceing! numir de much) Probleme de sapte st += 25 Se icc, ptr ore poliedru cae v vari, muchit s/o, axe loc egaltatea fovea? (elatia toi Euler 4) Dac mn 6 gi f = 4, determina», ecunonseb gi desea polieitul ‘Duck y= 12 gif =8, determina, reeunosstet i cesenat pied “26. Reprezentai in clesen dou pole, primal awd 8 vir s6 Fete 27, Anvelopa unei ming’ de flba! este confections in ical de piel de pentigouns reyulate seu henagoone egulste avin laurie de angie Se fie cain jor letra pentagon, se afé vine hexngeare Din eae bucst de pile este conecyionac anvelops ming ? dnd 6 vets gi 8 fee, lar al allen on) 5 | tices FINN Marius PERIANU + Oumirs SAVULESCU TESTE DE EVALUARE Subigctul «67 Désenay gi clenumitt corpul din figura afarurara, 3) Nuit mutate aerate ale corpus. <) NirmitieArfurile eare nu st situate Tp planwd (ABB) 4) Procizati auf total de Fee ale corps ©) Deterininai crcapta de intersectie» planelar (28) gi une) Sublectul I 4a) Desena) gi cenumifi carpal din Faure sles yy Nut toate planele care mrginese corpul oF Numigi toate petechile de wwehii necoplanare ate comput ) Precizat gi justfeati valoavea de adevir a propoiticis Daca We BC SiN @ AD, alune’ punctele 4. D,M 4i.¥ sunt coplanare 4 Timp de lueru 50 minute. Se accords 1 ponct din oficiu, vata? Subiectul | (1) Desenati sé demi eorpul din figura sltuarat. 1) Dati exenipla de o pereche de meh eaplansre 4) Dap) excmplu de tret wisluri care sunt situate pe aacoeas: fal a orp & Nurnifé ui plan leterminat de wei diniee varfurle corpului nesituate pe aceeasé fata «) Considerar» unt plan oa Dresieat suisse mis Sublectul Il. <2 varfri ale eorpla neituten plan cc in fia altuoit ABCD este pay, tar tanghiucile ADA, DCR, CBP si BAN sunt eehitsterale continate jin aeclagi plan, 4) Denamiticorpul care se objine pling trianghiveile chilotorade dupl creptele 4, DC, CB respestiv AB. 44) Numiti puneiele din figund care vor coincide dupa plier, © Baek 4B=5 em, caleulsli sume turer ewuchitlor computa cine 4) Peecsrat dstaota dintre punctele A/ i B. dupd plier. inp de Hueru 50 minute, Se a¢0rd 1 punct clin ofc ea. detexminat de ij dinke vrfrile corpului fet iat 2, apd abbas tah a, is pentru portofoliu Numele si prenumele: Giese Vilea—_ Parnes, drape, onan ether tat Complataté spayide punctate cx raspuneul enrrt 4) Trsi puncte situate pe aeons’ devapt se nuimese panel 4) Patra pumete nesinaate fn acelagi plan se numese puncte <) Pivami trunghigarh age o.oo Fe <4) prism ou tza hexnigon 30 numeste prism Pantry fiocare dintre enunturile urn stow adevarat, incarcws itera 8. fn raz conten ir <2) Tre punete necoliniare devermina un plan 8) Oricare dows puncte care sunt colina, 2) O piramida potrulaord are & ech 6 Toate fefle unui paralelipipedul dreptnghie sant dreptunghi cuit itera sa ereantiel este A 8 4 fincercuitt raspunsul corect fa figeare dintre urmstaarele overets a este rarseté Dintve cate patew variante de rAsaiins, dase 4 Nuuntuul diwpeelor care wee prin cei paneie necoliniage est A2 Ba et ns (6) Numdsal fepetor unui tetracdra este: a B2 23 + ©) Un pian este univ determinat de A-odrvapth 2.2 dree paralele 4) Prigena patraatert are PS infor. 6 varturi 4 2 puacre Soavertai 2 1 y8er Desonal 9 pramnids patrulaterd FABCD cu hava ABCD. 44) Neanips owchiile lore ac pir 2) Reslianyi o dosPigurere plan piramide! MARC. £4 puncte voliniare MATEMATICA clasaavilta ~¥ api 5. u SAVULESCU Mircea FIANU + Marios PERIANU = Desenahi 0 prism telunghiulard 4BC2B'C cu baer ABC wiunghi echilateral Feqele lateral ale prised sunt deeptunghir Se glie ef AB = Gem si A 4) Caleulati aria bavei ABC. 4) Calvolati surna etiuror nivelitor prisms Jo Tema Pozitiile relative a doua drepte in spativ Pozitiile relative ale unui punct fata de o dewapes Punctud apcorine deeptel. Be - Exemplu, i ed 4 c unc ameaportne dope (este exterior rope fxemple. Be, Cad relative ale unui punct fata de un plan ae Panes apertine plan ese 0h pan) aA Exemplu, 1 € ct. e is Punctul rr apertine planului (este exterior plarwlu Exempli. & ¢ «. relative a dowa drepte in spatit: Dou drepte sunt idenvice Ceanfuntive sau councics daca av dowd puncte comune Exemplu. Deel dce gi cca stunci de d is Dov drepte sunt conecwrene ac au un panet come Exempla, dre={0}- Lo ¢ Doui diepte sunt paratele does sunt coptanare si ous a puncte comune. empl. dea, ecagi dne=@, auntie. Bromplu. dca, ecayi dne=@,muncl dle, fe ous drepte sunt vecnplanare dack sent concureate gi nu sunt paralele Exempla. ¢ne=Osi de € ae bservatli 1. Se eonsidend dreplele ,# ge, Dav se 6 gi bile, atuinei =e saw alle (proprieiotea ee nzutvtae a celal de paraiso) 2, Boua deep sunt coplnare dacs numa daca sunt paraele si concurente 2 MATEMATICA Claes Villa —1 sey FIANY -Moviys PERIANU - Duritte SAVULESCU. co 1 ag) eryptet Ll vcapta 0B, 2h drop 4 kaa ale creapta DB: H aucpuci DB aya de dowapva ot Desenati o plramis pateulareed a) pavalel P) cone in Maura aldnarotd este sprezentat o piramids PBCDE ca on septal eu centr ‘a Reproducet dese pe eaiet 'b) Mengionati in seis pozifa relaniel a = cow chil laterals Sond nehti oareeare ale haves = dropleloe Aa 91D ~ pmoetnlul © ade deeapia 42 A) reptei 18 fd de dowapia ME, Hy dreplei NAT Gah de dreapta CD: 1 csoprel NC fata de dreapta 340, Ink BCD, Dati dow’ evemple de deepre me 6) necwplamare To tiguea alduaratd ese reprezentat parsledipipedal repwunghie ABCDARCD Precizati poritia relativa @ uematoazelor dvepie: a a) AB yi A'B B) BC $1 DC, ABCC: BDC, ©) OB AC, DAC INES BAC HAE WABI DC: yBes ae. iS Se considera punctele diferite A, 8, CsfD. Stabitti vsioaea de adevar a proporiticl Dac dieptele 4” si CB sue coplanare, atanei deepiele 4C'si BD sunt enplenare”, Jn figura uldturats, ABCD este plteat en centrut O, iat £ petele £ si F sunt situste de o parte side aa a planlai (AKG) astfel ineit panctul Q esty mijlecul segmentulud [PAL Arai ay puuictele A, 51 sunt caliniace: By punetele EC 1 F sunt caplanares ‘) pnetsle B. DE. F sunt coptanare 4) |DE |] BE| ins teuaedeul ABCD se notenva eu M,N. P sl sespoctiv & inifloavele muchiilor AR, BC. CD, Da vezi figuink Demonsirati ex AINPR este paralelogram. Fie tetiaediu ABCD si M,¥. PQ mijtoacele muchiiler 1C. AD, BC respectiy BD. Stabili dack: 10 droplele &P si 449 sunt coplaaare; Dp dreplole A4P si NQ sunt eaplavare ©) patrulaerul 44 PQ este paralelogram. Jn ede! ABCD se ytie e& AH = AC, Daca [AE sh (4F sunt bisectoarele utitor Avatar ca punctele Mf ¥, 0. ¥ sunt eoplanare 6) Fie puncwl & astiel inca Osa fe msijlocul sewrnentulul [JR], Arig: WHALRC, EB =RD: ti) FMITRN Se considerd puneiele necoplansre 4, B.C, D . Biscetoacen unghitlos #10 imerseetewzd segmensul [BC] in punotul Af iar bisectowen vnghiulsi 200 imerseeteaza segment 2D] fw punctal Demonstrated AAV || CD daca si mami da Z 2 5 Mircea FIANU + Marius PERIANU = Gunite SAVULESCU Tema =" Unghiul a dou drepte in spatiu, Drepte perpendiculare Unghiv! a doud drepte situate in acelasi plan. Se considers dove drepteo gi ‘onertremte in punetul Q. Cele deus dreple determing dou pereclit de unghiurt wpuse la vcFou misurite exsle cu v* respectiv 180° 4 4 Defnim anus nahi cdreptelnor i fe Bina a 2 in AN) oF ‘Observati 1. Duca er= 2 sau ails, ature’ erat} = 0° 2. Fie ad. sicfsuni drople aslel nest umvb 22, ead e@.atle st bid. Atel nb) = mi eZI)Aeorseci a teoremet nner ate neapeciv ple Unghiul a dous drepte necoplanare. Consierion dou depte nscale a 9d gi panel ty spatin, Dreple u? gi 2) sunt asc eit ale’, Bi)A* gh ebb = EA} Definim mdsnra maghiului dreptetora ay 1a mae Men su Meb Observapie: Se pate aly Definite. eh if sunt dou dente al Tes mi) = 90°, sun pte se umese purpuniculre. Naim @-L, 4, Desenaji cubul ABCDARCD Beternninarl masurile unghiului dintse deeptele: aw) DEY CD: by AA 3 BB Aa’ $i CC od) AB ¥i BE OABIADS — PHB si Ab @DCIAA, — MACSIBC DRC SAD. 2, Se considera cubul A2CDAB'CD' de laturd 4 4) Caleulag, ta funetie def, perimotrul whunghiulus 4'C"8: 4 Detecminati masura unghivlui dintae dreplele dB gi BC. oj Deremminali masuca uiiitalui dintre dreptete A'B gi AD’ 3. Desonati eubul A8CD4'B'C’D", Determinati Doh TOBE by BOL AC We o} 7a} CBO }unde O este mijlecul segmentului (A'C), ci figura lta este desonat un paraeinane oy vn ae DIS AP, ~ 7 gee ey reptunghie Precizali mssurs unighiulsi format de percchile de drepie: WAL IEC, — 6) ABGICC DABSICD: — 6 DDG BC, Paweaul ABCD si rombul ABMW sunt situate in ifeeCBO4 y= (10° gi mANM)= LOO, ene misurie dle dreptele: a ACS BD: Ace Mi DOG AN, DOCH RA sede b) aD si Bah 0) CE SLD. Sem si} Picaenida palace regulate VABCD are AB Determinati astra unghiuloi format de creptete: VAS YR BVA BC. gd IDSLAR, igramul ABCD, cu m(a DAB} = 60° si inipezal BEF incare BOUL si 10° sunt situate in plane dilerte, Calewlat¥ masurs anglhiatui Jor mat de dreptele a} ABSIER — D)ABSICE AD SEF Se consider patratal ABCD gi trupeail isuseel AREF cv E # (IC PCADE) = OG? Stind vad AP = LE = LC , determina misucite unghiuritor devplelor: WEP SOC WACKER PRB DEL Deeplunghiol ABCD gi uapezul drepunghic CHEF, iw cate COREE gi A AD SEF. mn(-£CDE} = 90%, sunt siete In plane tere, Se wa lie ed tse 2K9 = AP Si m( Fis& pentru portofoliul in Numele 5 prentanele: Clasa a Vita Poxigile relative a dows drepic issu aghiul adous drepre w sath. Drepte perpendicuitrs Compietati spatiile punctate Dumicu SAVULESCU Pantset 8 sum sien plan ae pine ese exer pn Pe si ¥ mioucetaseyoentlor [AC] respect [Ata 8 AZ be in eoehad ABCD, pneiee Ais ¥ sunt ailaacelesegmenteor {4}. respetv (AD) Ariat ed HC BCD) Fie punelenecoplanae 4,2 C4 punetle At B Rrsiloeele sxgmenieor [20]. toc) font eset [48 Arian cd RP | (BCD); NP BDAY gi MR YADCH. Thianghl 48C are lature (BC eons My plana ar panel este exo plaouli g. Segueatle [Z| 9 [C2], Be AC, Be AB sure mean i uianghil ABC. Deron of FDI. Dosenstio piraimida patrulaterd ABCD. Fie Af §1 M mijloacele muchilor [FA 51 LED. Previzay pozttia dreptel 4) AB taf de planul (ADC). by BC fey de planal (VAD ©) YB fag de planul (42C 4) 90 Pgh de plamul (480%: ed 4 ta de plamal (CI, BD) AW baa de planul BQ. ae Se vonsidert cubul ABCDAB CD", Notim ex ay ADCO; —BTDNCABCH: OM |LA'C'D) Incas. Se atte OB" DO" unde {0} =4°C D" Se considers prisnia uitnghiulené veyulath ABCA RC’ Punctele M,N, PQ sunt mijloacele segmenictor [48], [BC], [BCL respeciy [A'B. Se ste ch AB = (em iat 44" 3 Bom. a) Artie PQ ABC) by Stabilt natura patrelateenlui MPO: 1 Calewlayé perimerrul patrulsterulai ACPO. Jn weraedral ABCD, punenil £ este millocul segmental [CD], ar [BF 91 EC sunt Useivacele | onghiwilor —-cAED,reapectiv ve Fe A0),6 # [AC], Demons e& FG BCD) eenivare, Conform woremes bisecwari, 46. AE gy 3G AE pe” Gece verti es 26 28 Se apie apo seiproes teem Thales Oo GC pasos m Triunghiul ABC ave latura [BC Snelesa im planel a, iat ee. Se stie cb 4B~ Gem, AC =18em, iar punctele & si F sunt situate pe Teirile [48] si [4C} astfel neat AB =2em si AF = Gem, Justfieali ch EF Yer ‘Triunghiul isoseel ABC’ are baze [BC] inelus’ tn planut a. iar A ¢ er, Se stie 4B =60m, iar punctele E si F sunt sitnale pe Jatwile [4] $i [4C] astel inca om. Aritti cd EF ke = ACAD. Arita et Cum BF Se da pinainida PABC on BA = PR = FCS 1 ~ BC = AC, Se eonstauiese perpandicularels HO LR. 6 FBG APLOT, Ee CH Demonstrate HD | (BC Lesagonul regulat ABCOEF si phienul ABIAY sunt situa in pine diese 1a) Ce povije ore dreapea CP tata de plawul (14/12 vy 1D) AcStofi ch punctele C, FW, 44 sunt copkware We 6} Arata cf Greptele WE sk OW sunt coneuraoty Ce pozitie are dreapia #F fafa de plarul (CBM)? Bs {9 Caleulay rnisura unghiulsi format ele dhepecle MAN gh BA p Trapezul ABCD are boca «AB inclus® to pinal @, Nocim ex £ 341° mishoasste fatutilor AD respeciiv BC. Ce pozifie are dveapta EF fe cle plana ct? Puncti feste exterior planalui paratelegeamulet “WICH. Nokim cu & miplocul segmentului [44]. Demonsirati et CA | DEBI by Daci, in plus, AC = BD , ard ed WEL DE Fie piramida patrulsterd YABCD fn care nota cu £ gi Feentrke de greutae se feqelor PAD g respectic FBC. ArBlai of dreapta BF este porate wu bara cae Fic parlelipigedl dreptunghie ABCOAB CD" tw cave AZeu2 mijveal uch AA iar N este mijtacul muted CO! a Ardtay 8 MII ABC, 1) Caleulai lungimea sezmentului [MN] sitnd ch AB = 60 emsi BE = 25 em ) Dack 4’= 14 em ealculatiperintettul patulaterului a24vc. Siabitii valoaren de adevar s sieetreia dintre proporiiile: ‘aj Dacho dreapi este paraleld eu un pla, unc ce este panalels cw orice drenptt inelus in see plan. 1) Dacd 0 dvvaprd este parateld cu us plan, auc on este parafeld en o infiniate de sdrepte incluse ta ace! plan of ach doui drepte dierie sunt paralcle cu occas plan, wine cole dou dene sunt parafele Tare ele 1d Dock dreptele gi b sunt paralele Tntse ele gi dreapta a esto parslelt eu pla) fa atunci drenpla f este tnclusé th plane & © Pringrun punet exterior unui plan dat pues duce o infinite de drepte paral cu planul dat [p)Dacd drepcele agi 6 sunt accoplavare, stunci exist un plan a eare vontine dreapia a gi este paralel on dreapta 6 Prin varfurile criunghiului ABC tos drepte paralele Tnuy ele at ADL CO" Se gio ot 4” « (ABQ) Ad’ = BBY = CC" si toate punctele Av 8, sum situate de Access) parte a planwlui (ABC), Fie Uf si XN miloacele segimentetar [14] respectiv [BB 4, Demonstrgi ci: a) AIN|| BCH, 4) AB WECHNY CHOLABCIED: — dh ACHP, wae P este mijloea hi [°C 3 FIANU + Marius PERIANY + Du oe teed PARC si punetche LF = a, Aw (Wt, Pow (Castel nc iN WI sv) 48 YE amen ME PC a) NP | (ABC by A e480) Se considex’ eubul ABCD: BCD. Nomen (O}= AC BO. a) Demansirayi ed: (4 CB) | AO 4b) Dexeuiminagi (4° CRE DEB) beset cubul A8CD4 BCD! a) Betceninati (a! CAVA( DBC}: by Ariat en (CAD t'C# ©) Demonstrai ef diagonala AC inieupa plomul (CBD) teighitni BDC. In poisme wriongiiulica ABCH'E'C’ noram ev A 91 N miloacele muchiler [8C] si respect [88]. Demonsteati ci BC AMM: By MN || (CAB) a conwul de geeutate al Deepiunghius ABCD i tiunghinl MAB sunt in plane diferte, Pe segmentele [M2] si FAH] se considers punecole E si respentiv P asifel Incr ME=8 em, EA= Dom, MP =9 cot $1 MB 2 360m, Ariat ek LEY (MDC) Fie winnghinl dreptanghic ABC cu m-t4)=90° 42 = 36 em, AC = 48 em yi AN 1 punctle 87 daniel nat n7= 12 om iN € [AC asl tebe = a) Dack latura BC wiunghiului este paraleta cu un plan a stabilit) peztia éreptci Av fat ce plana ) Caleulat lungimite segmentelor NC, MOV si MC. ©) Caleulal petinetral Givaghiulas BAN. Un plan este paralel eu dows fatur ale unui triongi. Ardtafi elt planul este paral 9) cha tei [oturd a trunghiului Fie d, 8, ©, 2 patra puncte necoplanare gi Me [AB], Prin AV se constiegte un jolan castle incit 0 He gh. | CB, sar intersestenza creptele BD, DC si C+ speciiv in punlele MP gl A Nelans AM = © en gi se cunoss 42 2D = Iem si C8 = 15 em. Exprimali perimetcul pateulaterului MAPR, Daed o dreaplt a este perpendicular’ pe toate dieptele ineluse intun plan @. atunei dreapta se numeste deep peepenndicntard pe plane! e Nowiim «f Le sau @ id on | e owt 103i b deus drepte concurente gio dreapta dL (en) fence tunel d La. i Dace dreapia vse inclust in planut faz 64 ancl d Le avs deste 0 dreamt i 4 este un punctin spatiu, atunei exist un uni plan ex care eanjine punctul 5/ este perpendicular pe seespta if. acd a. este un plan s34 este wn punct io spats, anne! exist 6 unied ddreapra d care gontiae pineiul 4 gi este pexpendiculara pe plano. © Droptole care comin punetad 4, diferice de perpendicular pein 4 pe planul ce, se numese oftice ineapott eu e YSeoh Fie @ este un plan, fun pune in spapu gi 4° pleionsl perpendliculacei din planul e Lungimea seementului Ad’ se mumeste distant de Ia ponctul 4 lapanul e. Notam dts) 4a Oheorentio Daed punctul d exterior planulul ce, dreapta AA", A’e a, este perpendicular pe plamul a, ir deeapa 42. Bea, exe o cole otear, ane) AB > it r vate Se easier 6 pramids eu vital F 9 baa 4 Ng inclustn panel @ Pi O picionl perendialasi sin peg Koofm nS Plael [3 2 ‘Segmental [70] se numeste Aniline piranue | g Nottm fa PO wath “ 5 7 Migcea FIANU + aarus PERIAN « Duritru SAVULESCU i ' Te Desemati eubul ABCD4‘BCD" $ indies 4) uw chops pexpendulaee pe planal (ABC: 2) wei drepie perpendieulare pe plaral ¢2D0 9 «) out drepte perpendiulare pe planul (t') 4) tel drepts pompendiculare pe planed (2'2°C°) fn figura teuratd este reprezeutat cubnil ABCAEPGN, lustiticti ca DEAL BC) — DBL BA ACL HD: DDB LUC, Se consileed parslelagramal ABCD en central O. De seeeasi parte 8 planulei (ABC) se diod " porpendicularsle M4 L (ARC) 93 NEL (ABC) astfel | incat A44=18 em gi NC=I2 em. Punctul F este | c eniloenl seemeni [448 (veri flan 0) Ardiai ch OE 1 (ARC); 4 4) Determinatilungimes segment [OE Reprezentay yerpendiculew 847 pe phanul . Dae A8-=6em si B Reprereotsfi in desen ws plan ea wn tringhi echifareral AGC ineclus ta a si perpendicolara CP pe plant a. 4 Jusiiieat ed CPL AB @) Doct AB =Sem si CP Desens um pitrat ABCD de eonsra O si perpendiculara OM pe plana paral Wy Arti ch A= MP = MC = MD. B) Daca MA= 48 = 6, calelati OO Reprezentai tw desen un drephinghi ABCD de entra O si punetul ¥ exterior planulai (AiC) astelincit V4 = 7B = VC = PD @ Asitatie& VOL AC b) Arial 0 YO. (ACD): ©) Dach FA= 48 = Gems) BC = 205 .caleuat oi{¥{40C)) Se considera punewl / io externa) planului pitaului ABCD astel ined 1A = PB=VC = VD. Notira cu 4) = ACM BD. Demonstrati cd 10 L (ABC. Jn varfil € al wiunghials} dreptunghie 4BC cu m4) © 90" se cide perpen siculara CM pe planaltrunghiulus, Artist ot BA L (MAC Se considers patratul ABCD cu 4B = 4 em, In punetol 4 se vdie’ perpendieulara ‘A¥f pe pianul patratulus Se sie ck 444 = 4m, a Calcul AC; 2) Justitia eb wiunghinl WAC este drepeungic, ce) Calovlayi MC. 4) Wesiticatidsca vigehiu) AMBC este drepeunghic. Pe plan phieatulai ABCD se ried perpendculats 44, Stind cf AB ~ 6 em si aM = 32 em, ealeuleti: a) dt, AD BY ACC MAB} ck, (MAC). n desen un plan c, punciele 4 si 8 situate in planul o em, calcula aah, 5) Aviat el PA= PB: 2 em, ealeulosi Pa Pe plana) drepunghivioi ABCD se cidied porpendicutas Mh se ste AB = hein, BO= |S.om iartN = 3m. ip Caleulay. Hits MD: 7 Coleutay kA BDYSE ADA: eseel ABC se rich perpendicular IP Se gtie ot 4p =8em, 1) Daca MY este mijlooul later [C | ealeula P40 lag at 5 Pe plepul riunghiului AB = AC =6em, iar RC wy Caleslau PB: ep Artin ed BCA GMP DC 14, Pe plagul tunghiuks’ drepunghie ARC se ridicd perpendiculara 44 Se pie et AB =d4.om, AM =Gom, iar oi 4) distanga de la punt B la planul (AMR, ‘9 distant de [a punetul O la dreapta Mi Pe planul sciunghiulul echiloteral ABC cu 4B ~ 8 em se ridion perpendicularcle CC's BM eu CC’= 12cm. Caleulat lunges 88” sstiel ined a) wimg 42°C" 58 fe dreplunghie in C's 2) triunghiul 48°C" 38 ie isoscel ou BC" = BY Considerdn trivaghiul schilateral ABC’ cu lauua de Mem In punctele © yi B se tides perpendiculanele CC” si BB" pe planul tC), Punctele C” si 2” st siete deo parte side aaa planului (480) astfl ines BB’ = CC’ =x > 0 Determnati caste) incat wiunghiul B'C's6 fe drepeamahie. Fie tiungbiul ioseel 48C cu AB © AC = Mem gi BC=36 em gl D millocal Jaturii AC, in punctul se ridicd perpendicular AM = 10 em pe plant (AAC). 4) Calculailurgimile M8 5) MD. 1B) Fie [AF 1 [AL bisccroarcle unghiurilos MAB gi AAC. BEB]. F e[WiCl Aritat eh BF 1480) 6) ColeutatiJongimes segmental [£4 Dintran punt O exterior unui plan se cue petpendicularcle OP, OF. OF pe ddropiele diferie 8¢% C4, AM cantinute in ar. Demenstrati retin BDDC +C8? Ba + AR FB = Seconsidert cubal ABCDA'R'C'D’. Dack ACBD = {0}, int punctal Af este contrul de grevtate al trimghiului BC". srétali ca DOL (AMC) e318 fe EVALIARE tn Figura alsueata este wprevemtnid © prisms drenpe fn care bass BC este tsiumghi echilaterel. Nothmn ct Mf ‘mijloewt seamentului [BC]. Justified cf: a AB ICME By AM! L480) oP AMINABC Ys dh AM LBC BCL AA {In figura atacurat este reprezentata opiramid cw tava pital ABCD si cu toate muchiile congrvene Nota eu ante baz a) Ustiicas cd BC|I SAD): +B) Armatiea FOL (ABC): 6) Dock Meste mijloeul chi [VA a8 8 ¥CIIDB) iy AV (bb) ‘Timp de luems 50 usinute, Se acord | punet din oftciu Hltces FIANU - Macius PERIANU + Dumitiu SAVULESCU tet in Figura Se ste va I= 12em, AC oy ssi ef FC" MBO) uy by dosti AB L(BCC) we ©) Ardea ch 18. BC @Cateulays BCs e) Caleulati ac. 2 Sn Figura alta exe rsprezentat ttre! ARCD in care rangi) AC este echilateral iar temnghivle DAB, DEC sh DCA sunt isoscele $3 Uroptuoghice In D. Se gue 2 ch AB = Gom, A 4) Justlicap AD L (BCD) fi 4) Aritatied BCL AD. Le @) Caleulagi AWs1ABC) 4 Caleulati et AB) Timp de luests $0 minute, Se acorda f panes dia ofici, Testal 3 - Compleat spifle punctate gin ermivoneleafiensi ast net 8 obinoi nui corset: au Enea one paride este 6) inaltimen amet prise este Drepiele a si B, concurente th puncial Mf exterior planulu 04, intersectea plant oan puncicle 4 respectiv 2 4a) Dernansirai of dospta 18 este inclust th planul 0: 4} Daca purctele D 31 & sual mijloacele segmentelor [Met] respectiv [4/2] argtagi ci DE a. ©) Dacd Pe (MA). Qe (MB). cu MP = 4 em, Pa = 12m, MQ- Tom ¥ G0 =21 cm, stabi pozitia dreptei PO fata de planul a. a Pe dheptcle « 31 se consider punctele 4" si respectiy astel inci A438 fe rmijlocul segmentelor 44°45 BB” Stabilité poziti deeptei 4'8" foe de planal =.Piramida patyulaterd eepuloré VAMCD are toate muchile eu lungimea de 4 em, 4) Calculatiavia bazei, 2) Calculayi apoteina piramidei Gslyimea use: Fe ators). ep Caleulatt ACP) 1 Damonsirati ca CV | UBD} unde A/ este mijloct muchiei (VA) vas Timp de hee 50 minute. Se neordle | punet din oie, Fisa pentru portofoliul individual Numele siprenumele -G [irsopta nerpeadirutnnd so Cistanda lela an planet fe gone: teeHiosen ort ee ipletati spetilte punetts cu réegniesed © ese “aj O despis care ow are niciun punet comun Cu um plan esse ce plane! bb) Daca o dreapta are cond punete camune eu um plan, tunes en este in plan ) Distanta de la viru amet piruide We planal buzei se nuneste piromide. <) Machi ltorale ale una cub sunt su pe hazete cub Pentra Gecare dintre squne seewial, fae ahs a) Dac o heap este perpendicular pew deep dieu plan, azunei ea este perpenciealara pe acel plan 1) Duck o dieapts este parsteld cu o dreapta dinte-un plan, tune’ ea este paral cu pian! @) Dac 0 dreapes este parale#8 cu ut plan, acunei ea este ‘pavateld cw toate droptele din ace pla 4 Dec’ dove drepte sunt peeperdiculere pe weelasi pi, atunei ele sunt parle. kt cake pn sebaiesb fap Numicul drepeclor énse peintrun punet esterion unui pin. paralcle cw plaul. este AL © ingsiume a2 3 L 1) Nuarul dveptelor duse print-un panct eslerior unei dept. parsiele ew cen reap, este At 2 3 Ls O falitate oy Numarul drepichor duse printers putot exterior unui plan, purulele cue reap data din acel plan, este g " “3 5. O infinite 2 Al He? © 3 <0 Naaru! dueptclor use print-un pune esterior unt plan, perpendiculare pe acel plan, este" g Al B2 oO 2,0 bvinitate # 2 Te Descial paralelipipedul dreptunghie ABCDA'B'CD. Se curose: 1 om. a) Andale AB LEBCCY Dy Arata eh AA YBCO? a Caleulath AC dj Caleuiati 4° Caleulal) dA WBOD'p (ls PERIANU - Dumitnu SAVULESCU imp de lucts $0 minete. Se aeords 1 punt din ofc, Tema 2.8 Pozitiile relative a doua si trei plane. Plane paralele. Teoreme de paralelism A.plane we cui B. plane seem plume pratt > — o1____- {BF / 7 notim a= ape nomi af = oti a fe Observatie. «= # deci si nunai dacd ra B yi Boo Lema Fie pleneie a si fi si drepicle «yi &, oa $i Of. Dack asi A atonci ol) lh Conditie suficienta de paralelism » dous plane Doug plane sunt paralele daci una! dinire ele conliae dows drepte concerente paralele ex eel de-al doilea plan. ‘Teoreme de paralelism Teorema. Fie planele a, B si 7. Dack alls ai any =d, ancl exisin dreapta e astfel incdt Sry =e iar die Teoremd. Fie planele poneie nf si deopelé paraleled/ sie. Bach over =f ated exit puncte B.C $8 D ostel ett laden. Where si 11=e08 In pls, (48) [10C] «Bout plane poraleledotermine pe dou dete paatee pe ear fe intersted, segmenta eongraente) as Teorema (tranvitivitatea relaie! de parsletism) Fie pianelow. Bsiy. Dacd ail? i Billys atunci yl a .- a (oud plane paratele cum al tciles plan sunt paratele nie ele, Teorema, Fic a si # dova plane paralele si punctele Ae. Be A. 5 Dock AB La, atunci AB LS 3 Ta conde coreme’ de mai sus, lungimes segment a se mumeste divans = inte planele §i ff, Noldm AB = alas). z Feorema lui Thales in spat = Teei saa mai multe plane persiele determina pe doua drepte varecare, pe eae eH Intesecter7, segment proportionate = -Mirces FIANU + Marius PERIANU - Dutra SAVULESCU 3. 5 Devwhizind o cane se abfine u euntigeratie cain lesen aftnurat. Preetzap gi demonstrat poz planclor (10%) $i 4ROUN, Fropecal ABB’ gi paralelogramel BCC sunt situate in plane diferite, Stabibyt poziia relat ploneter (4207 gi (48°C. in cave: a) AB x8 bp a Ba Desenati cuba ABCDA'BC'D™ ay Justticas v8 (ADD')||CBCC) By Aston ¢) Dack AC' = 2803 cm, calculati distanta dintre dowd fee apuse ale cubulus « (si nukim eu MA PG — ck be AFL BOCK: Se considera piramida HABCD cu baza pilratul ABC imijloacele muscillor ltorale #4, 72, CF cespectty VD. 4 Stabili. poziia planelor (480) gi WNP) 15) Deteninafi miportl perimetrelor patrulatetelor AANPQ si ABCD. in exteriorul pitratalui BCD se considera punctul Af axel inet Ma = SME = AMD. Un plan cparatel cu plasul (AC) tae segmentele [Mid] tno! [258] awed si ct we in B LMC] in C* si [AID] 1 DY Gtind ck AB=2a d(As\ARC)) = 2, cate distin dime plaele @ i (4BC) Fe prsma dteapd ARCDEPA'R'C DEF” t cane bara este hexauonn! regula ABCDEF wert gora ature). Arita eB a) (AEF YCRD'y ty (BEL HIG’) Manele (EE. (ALD gi (CFE) amo deapt comunt a Dac (HEC (DEE') = anne fll DB! 2) Pach 18 = 4. caleula d(( AFF} {COE)) Se considera poilelogramele ABCD si ABEP situate in plane difarie eu ventrele O si rspectiv OF aaa @ (BECIARD) by OO FD RE 6 20" I(BEC) Fie ACD un wunedru $1 puictele £6448). F e LAC) i Ge (AD) asttel inva 4é | aK ae URC Prin virile iunaliulul ABC tree dreptele paralele 4H 8B" CC 1 a 31 C788 Be de aeoeas! parte a planuiul wanghinlui ABC. Fie G yi feontrele de yieinate ale tiunghivrilor ABC si rospeetiv 42°C", Novim ew Msi i nentelor [12] respectiv [4°6'], Avago Acttati ot (EPG) [CDI fel inet rijloacele 8 a AN BBS bp Punctele CAMA sunt eoplunares oF GG" | Ada 10, Triunghiul dreptunghie ABC{ nt) ~90° I yi drepuunghivt Aus44 sun siuaee 19 plane dferite, Stabilij pozitia deepte’ 28 rap de pal (16Y). 13. Peoeizai gi justiioat valoares de acevtea prepozitil fa) Deed douk plane sunt paralels, mune’ eview drenptd eaniesd imteunul dint plane este pacalels ou eeallt plan. fy Does dowd plane stint parslele, tunel orice dreapta eonyinie Toten nse plane este parle eu ininitate de drepte conte in ell ph Ey Dach dbl plane su paraele, une! orice dieapté eva tne irs plane este paraels cu orice dreapts contnuta fh stall pln { Date find planele nepnratele cr si $5 un punet Min spajin, AV ea, Me ft ‘xisa 0 unicd dreapi cl ear eontne poset AF seste paral cu plane «1 ) Date ind planele a si A exit o dreapt a, a er ose net ail [f Date Fins dow drepse necoplonary of yi punctal Me Mea. Meh aise un plan cace conte punctual AF yi este paral cu depts $ J Dact Sona plane diferite sum perpendiculnre pe neveasi dreapt®. atu eeks dou plane sun. paralee ate cl fy Fate sucio! ca doua plane 38 Tie paalele cu aveeag Jeeape pent ca ese dou plane 3 Fe parle Inte ete. {) Dacd dont plane diferite sunt parsicle eu dou drepte necoplanare. sinei cele doua plane sunt prnise 42, Punctele 4. B, C si D sunt necoplanate, Fie MLN gi P mioncete seynientelr {ABLLACI si eospectiv [4D] 4) Acttat ef (WNP)|I (BCD 2b) Acitaf ed wumghierile ANP 5) BCD sunt aserene. {2 Caleul valoarea raporlui pevimenrelur wiunghivror USP 9.220. 2) Calculativaloarescapersulus anil wiunghiarior ANP SE BCP unc 18, Punetele 4, 8, € gi 2 sun, wecoplunane, Pe Li 8 gi P mijloacole [BC], (CD) gi respectiv [DE], iar G,, G, $i G, eanteelo de Uiuaghiaelor ABC, ACD si sespectiv ADE. ay Acitagi ok GC, ||(BCDY «¢) Caleulati valoares eaportulut perimeuetor tiunghiusilor G.G.G, yi BCD, ) Calculati valosres raportula avilor tiunghiceilor G.,G, si #CP. 14, Se consider’ punciele necaplanare A. B. Csi 2 $1 punctul 8% A 6 (40) a, care confine punctul J gi este paralet vu drepiele a 54 CB, in dreptele D8, BC's CA in punciele Psi sespestiy Q, Aratay ed a) MY ABS Wp PBC: 6) SAPQ este parale d) AM-CP= OM BP. Se consideré paralelipipedul ABCVI#' CD in eae O yf.0" sun evntreke harelor ABCD i respocite 4'B°C'D" (o Acktawi cB ('BD4|ICCB’D'Y 1) Dermonstint oF (G.E,0,)I1 BED) Plan by Duterminay) WICC MECHA. t 5 MATENATICA Canis 1 SAVULESCU 6) Dyoa 10" 4 BDI = 16 A¢CRID) ~ fC} erno 8 prmnetete G, si Gi, sont veniele de greutae ale wianghiueilor 4°RD givespectiv CBD". Aritalich 46,360.60 ‘Se vonsiierdeubul 4BCDABCD" in eaee a9 Attati ch (2'BDIII(CE'D 1: 1) Acatalica AC’ L(CA'D': ej Cael a (78D) (CRD) Se considerd cubul ABCD.4’B'C'D" in care AB = a. Se noteavd cu MNP. RS $i Fnijloueste machilor [4D}, [42], [°C [C'. (BC] si espectiv (DC) 4) Arita c& potzlateral UNPH ese dreplanghi By Avttanj oh PSL (ABC): 9 Cateulayi d( (PS) (TAN) baci six sunt doud dropte necoplanare, ardtati cd exist un plan si mural wu! aves evi pe una din ele gis fe partel ew weatal kak Se consider plonele paraicle $i #8 si punctele 4. 8, C si B astiel inet AB ea gi DCE f. Punctele Ms N sunt situate pe seumentele (4D) 31 (BC. pact 4 288 oritali ed aN la MD" NC Se considers planele a if asifel ineke am B= gi planul 7, y lta Daca NB Se considesl planele & gi fi sidloua deme w si a Casi be B. Duck off, yoasesi ddemonstrai ef all> iva si off} acta ed planele cgi 8 sunt paralete Probleme de sapte stele Se considevf planele er /f.y- pavalele inte ele, Punetele «i & sunt situate tn plavul a iar punelcle Csi £2 sunt sitvate in planul #. Fie LM P= aCe, Vy atieiy 4 ICO Acai cd, doet punctele A.B. € si sunt nocoplonere, atuci punctele XP si Q sunt varfurile unui aralelogran. (Analzat sivabile posible dasa pile eek iret plane) Se consider semidreptofe necaplanace (Ox, [2p si [= gi punetele 4,B« (Ox, ede si Dash [M}=ACABD . {NV=CENDP §i (Pt = Al c\ FB oiog en punctele Af, N, 1 P sunt coliniare. Desergues) Se considers relmaednd ABCD si punctele M eCABAN LBD PED) gi QatCA. Araay = BDevp si 32h =( ponctele UY, &%, P si Q sant coplanare daca si numai aac Tema ©. Sectiuni paralete cu baza in corpurile studiate. Trunchiul de piramica fin gurile altturare este neprezentals sectiunes realizaté falro prisms de un plaa er patel cu hazel Corpunile eezuliote sunt prisme de acelagi nume cu eis inipal, Definitie. Dintavta dinbe plancle cclor dona baze ale unei prisme se mumeste name prisms, ‘Observatie, Lungimea muchiei lateeale a unei prisme drepte este esta cu ndljinea prism. in figurile altturawe este reprezentats sectianea realists intr ‘prams de un plan «& paca os bara, ‘Cole dout corpuri re7ullate suet 6 pirawids azemenea cu piramida initials gi us aemei de pivonai Definite. O pani eare ene ca baz ut poligen rgulat si a efret iahime conne centul bazei se mumeste Pika reat atin. Segment deexniat de vst une! piemice ulate loc une Befintie. iste ie pele lor dost haze ale vou neh de piri suet Salinea enekin Deficits. inan teancht de plromidd regent, segmental determina de nijloacele bazelor uel fe tera se nomegle open o crcl Sectiuni diagonals Defiritie. Sectiunea realizath intr-o prison de un plan daterminat de dou’ mshi Imerale care aus sunt inclnge in sceeasi FAA lateral a prismet se mameste sechivae @ MOB ID BC cadeneteos ddagger a rime g MAND PC Nt Definite. Scctiuten eliza tao piramid de varful piramidel io diagonalaa bazei pita se mmeys seme aogoes a pram i Mircea FIANY » Moris PERIANU 0: Mircea FIANU + Merlas PERIANU + Gutta SAVULESCU * Se considers psi riunghiularasegulata IBCH'8'C" ty eave Ketele Iker sat pte Planul este parsley bazole prismel gf inerset (ed) (BBY $i (CC) in panetcle A, BY yi aspectiv C* A Semi Ad” = Som. 4 Calcul report oalimilor peismelor AHCA"BNCY gi ABYC ABCs 2) Caleulay sapere dinte ara uned Hee Itorole & prise! ABCA B"C™ shania une! fete lateral a prism’ °°" °C” ©) Caleulti port inte sunsu lungimilor tncwar enuchillor prismei "ACA" B°C* seni lungimilor tauror machi pissed A'B'CTANETC™ Se considers cubul ABCDATH'CD! In eate O,, O., O, $1. O, sunt censrle fxcor locale AAR, BBICC, CCD'D, especiiv DD. Seslieet 48 = Sem by aatapea 0, = 26, val muchiile knerale Se slie oi 4) Caleutesi AC ©) Arata cd paireleterut 2,0,0,0, este paral: Calcalatiavia patrulsterului 0.0.0, Prisma pattulalert regula ABCD4’B'CD’ se sectioncnz’ eu un plan (EPG) , cece), poralel cu planul (ABC), Se cunose Fe(Ad),F e(BR) 4B Dem, Ad tat patrulatenulai ABCD: ) Caleulafi pocimercu! pacrulsterulai ARBRE 1) Calevlayi perimeteut teiunghiuli BEG 4) Caleula ar Phamién patrultera VABCD din figura sldtureth este soctionat’ ew un plan @ paralel eu planul bazel, determinand sevjimen ABCD” va a) Lustifieas ch 478 48, Hy Artcni oa A 5 ” i WAR, A) Arti = Gem, aflati Tunginiea muchiei [Fh ae ©) Dac AB = Wem, 4°8" Planal @, ail{4BC) punetele 4” € WA, BY € PB §1C"€ CV. Demonstrate: th VDC! OBC va WR VC AB aC BC 45) Teiunghiurile 4°B°C" i ABC suwt asemenee. Generalizai! coogi dd! liersectea2 muchiile Interale ale wetraedrulul #48C tn - Pe muchiile (4, (HY) gi (CH) ale piremidei wiunghivlare VABC se considers WD _2 ¥E ponctele B. £ si nexpectiv Patel nett “2a 2, PE? gp , wren Sa 7 B75 © rr 1) Plancle (DEF) 5 C422 sunt parle 2 FL scans 4 ekg) DEW AB: 7. Pe muchia (W21) a piramide! VABC se eonsiveré punciul B gilel Welt 5 3 iar prin D se duce planul paroled cu (HAC) eave take anuet ile CI) gi CED in porctcle gi cespeetiv’ F. fp Domonsira cd whungh utile ABC $1 BEM sunt asemnenen: £9 Caleulyirapovtal perunetrelor sé apart wider bianghiorilar HC si DE 4, Piramida prinaincer’ regalats ABCD ave fa) Acatay eb Vol PB = FC = ED by Pein purtul O° -miocut talc picamide, se due plana! pavslel eu ei. Stine cf 45 ~ Mem, eWICUlat ars seetunldeterminave de ima FO, pawl be vplamal @ in piramida, we 9.0 pixamida wiunghiularh regulatd KABC are baa ARC de centr are AB = lcm si VA=10-em., Fie A mijlocul muchie’ [BC a) Calclati P, 1) Se considers plane paral eu (BC) sf cane cont sacfiuit obginute Tn pirenida FABC. 40. Pe fualgimen FO 2 picamidel cegulale VARCD se consider’ punetalS, Prin puncul Sse duce planul paralel cu plat) bazel piramide; care tae machiile [74], [75]. [CV LDL in punetele 1”, BY, Cosi respectiv Sind oh AB=Uhem, 7O=!2em gi ¥S=4em, calcula a ae, 14. Se comsiderd prisons patrlaterd regula A8CDAB'C'D! To care A =9en si ‘88'=12 em. Pe segmentul [BC'] $2 sonsiders punctsle MF si AT astiel inc t= MN = NC". Planele a 34 6 sunt pavalel cu bazele prisme far AF ea $1 Nef Caleua a) Lsoginsea segmetului (8C']; 6) lar). 5.S¢ eousiders cubul ARCDAD'C" Punetele A. 87 yO" sunt situate iuchile (A4'), (BB) girespectiv (CC' ati ined Ad" Ba" = CC 4) Aeitayc&."B°) ABs bj silicate C2 BCD AREY, ‘yn 00" eritaica 2 = PE . ea DD 13. Se eonsidesd pasalelipipedul Grepamghic A8CD4'B'CD" in care O,. Ops O, $i 0, sunteentelefetor laterals 44°8'R, BBTC'C, CC'D'D respeetiv DDL a) Aesiaji 08 00, | ACs by Ardtaji cd (O,0,0,)1ABCY: {0 Caloulai raportl arilorpatulsterelor (0.0.0, $i ABCD. pateilipipedulud A8CDABC'D" so comsiderd puncte ppunctul Calcul avis fy Langimes apoteme’ piramide’ M2. oJ Dac {") 14, Pe muchiile laerale ale o 2 s SAVULESCU (iroua FIANU + Marius PERIANY « Our we Prisina hexagonal regula ARCDEPA'B'CD EP" are AB sconlannre 3.8. $i Q. Demonstrai ef punctele A, N, # gi Q sent véefurite aml parllogra, Se consicera prisms prirulatera regutasa 4BCDA°R ay Jusuiieali ca Ce (AAC) B) Daw AB =1BoM,I8? Agee = Ay, Sect peat cu louis do 63 em, Calelat aac Wy ae) ©) vin unei fee lateral, Se consiesti pavalelipipedu dreptunghic 42CDA'W'C'D" in care AB =4 em, BC = Sm i BA'= bem, Caleulay: DAs WD Avi — 8 Argon Dow baa patratal ASCH, » calcula ntlyimea prismel ae ABCD'8 CD! este un tiygonalt 10.0" prismed patruetene 5 4! = 2em. 4) De terminati mamarul de seotiuni diagonale nevchivalente ale prismets #9 Coleulatiarileseetiunilor diagonal alo prises. Un tunchi de pitanidi patsulecera regulara sre fongimile iatuilar bazelor de Iz em gi 8 em tar apotema trunchiului este de 6 em. Determinaji mariemea {naljina peramidei din eave proving qunchiui wee Un trunehi de piramids patrulotera regulaid are aria bazsi ici egal eu 00 om? si arin bazei nstri $00 em’. Se face © secjiune ou ua plan paralel cu bazele la acewasi istan\a fog Ge ambele haze, Cateulai aria sect Un trumehi de piramida triungliular segulata ore intljimen eget cu 12 em, apoterna bazei mari egal u 6 cm gt spatema haze’ miei egald eu 4 em, Caleulaf: a4 ljimen piran ide: din cave previne wunchink, #4) Cungimea apetenve’trunelialu de pirsenida Se considera im truneh de piramid potralatera regutatd cece are hungimea faci Ssxzei wil eg hingimes lanusii bazet miei wala ex J 4) Aritati c& punetele de inlersectie a diagonaletor eelor patru fete laterale ale lvunetalut sunt situate gran plan a 49 Exprinati nea seo(iunit realizate de plaml a th tanehial de piramias dt Probleme de sapte stele Betermini mamarul de seetiunidiagonale ale unei piranide a) tingle, 0) hexagonale cousier8 0 piramidi patrulater3 care are toate emuchiile cangruente. Arita ca jonen Giagenalaa piramide’ este un tangs dreptanghic Seejiuoea diggonula & nei piramide panulstere regulate este wn tciunghi ectiliteral cu locura de unginae - Expr: 1) Lunginsea muchiei buzei pizaindel: 6) Lungimea apotemsi piramidel op: copy api fp) 1p) Op: ne! ae! NOTA, ap) i Noa, Festudl 4 in Figura altura este mpezentala o prsent deeaptt in cate Buea BC este tciunghi echitateral. Foe 4, M51 P mijloacele muchilor [4B [C] si [°C @ bustficay ok (BCI ABCD ) Arba cd NPIUBCC'Y ’ 6) Acai ca MICE & Ariat 8 (ANP) (BCC) 6) Dich AB = Bom, calculali JULMNPK(BCC). A In Figur letra este enrezeniaiao pirmidh cu been pnetsl ABCD. Naver ou M,N. gi Pcomte de greutate ale rinmghiarlor #48, VBC $i CD 4 Lasitiat cd WAC by Anton c8 (MNP (ABC) 9 Caloulayh A «0 Dicken piramidei este FO=9.em, ealevlai (MNP (ABC Timp de tuera SO minote. Se acordi | puret din otic stl 2 fn figuc alsa este rgcezenta un paallipiped deptunghie. Ponsele 2 OF sunt conc bazlor ABCD. respectiv A'9'CD! 4} hosient es CHCAB'C) 4 botfcal ob (4'€°D)|I¢ 48°C © Arita eh ODO BC 7} ealeulai 2S; 1G}, caleulati op Dach #0. BD" ©) Arita punetal G este central de greulate al triunghivalui 48°C Ip Figura albnuratt este reprezentat un teunchi de piramids wiunghilacd cu inifimea #=9em, AB=6em si A°8 = 4em, 4 ce ec 7 2 Calealati SE 4 CE ; Sc ) Casa Trayimes pivamide’ dia care provine i rennelil Finny de luera $0 minute. Se seord | punet dis otic |asruench es isa ortofoliul individual Tesial Figa pentru Pp a) Numirul 1uluror muchitlor unui paralelipiped dreptunghie este Clasa a Vil-a—— 340) 3. Dreplunghiul ABCD de centre O are 48 = 4 em, RC = a3 cm, in vartul (2p) 1. Completatt spatiile punctate cu raspunsul curect: 14 Down plane care ou 0 deca comua se numese plane > Doni plane eare on au nici pune eomun se nomese plane 2) Distaaya dintre planele celor dova ave ale nei peismee se oumeste W) Nil EO 1 (ABO. by Demonstali ed (EOF) [BER ernie rise | . 9 Caleulatihurgimes BN, G Segmental determinae é¢ mijlowvele bizar unui wrunebi de: pital spl 4 Fie piramida patculstord reulsti VABCD eu AB = 20 om si PA = 26 om. : regula se nue oe inne owt cu M91 P miloaeale muchillor 48, 18 respect BC @) Cake ara tiarghiull Pd Ete adeviratsatrmatia (WAC) (MANDY? Juste ©) Coleutag porimetal tianabilui MNP. 4 Calculayi distanya dela vartul C ta muchia #4 HOA. Timp de fuera $0 minute. Se aeord | punet din ofeiu ap) 2. Pentru fecare dintre enunturile wimatoace, daed wats! adevarat, incorculti ites 8.1 caz conteat incencuct its 4a) Died dout plone sunt povaele. atune! oviee dreapra comnuld inu-unul dinte plane exe poralelé su eelabt plan }) Ducd dout plane difecite sunt perpendiculare pe aoseasi reapti, ature cele dou plane suat paralete inte el. 60 prism’ dreupla cace are bua un peligon regulal se nurnegte twiraeru regulate D Longimes muchiel Interale a une prise Ueepte exe gold eu langinsea Snag prism Yestul 4 Incereuiti singural easpuns vareet: engenders 49 2 Ah BD oe pals pl 4% . 2. Uno pt in sg foi 27 de abled oe abe sun picts pe ingur fa? 3) ce abn sum pits edu foe” 2) Ge nb su pn eee 4. Recut A0CDEMGH ei lee de Go Tell aff puna ser jung nunc unde st Dr eine © eet pnp aed TEHE. a oun taviehpe mas ie int ta foes pe se IC Fretpuind cb cone ole mer cv Sven ees Mal ete Tare shina mina setae urge dec? 5. Dr opt bate st ed ea couse an eb a musa de 3 tn. Niele sn in ec se leah tale mt le putcaves) mule Scere tng min atl 6.0 ssl ted un eb een 48047 machin de Deo npn Sian Pal sepmont pecs este o eae dn epmeru [2th unde cscs much {7, Uintolseqat sep cu pul BO se cpa Nye mucin CCL ul eget capital hn Sevincy onc sacs méndnct 3 tan de eva, x tase ee eon ‘Muda ol cucsrostaie Daub a) Po G, $1 Gh contrsle de gyeutate ale wiunghiurilor P42 3i PRC: Aritati ea GG. Jac 3 ‘etn yams 2 este seprezencat sebentatic un cort din . pape’. Contul geie 0 priawa dept ew baat un Arunahi cetera, ce aioe 2m giatura bazei de » 18 dim a) Determinati ciji met pateats de paz sunt . necesari pent coufelionstes cortulu? Goate fecle ort sum din pine’, inelusiy podeais) 2 Deterninati masura unghiulul dniredrepicle AC si DE. 2. 0 incapere ane forms una ew ABCDAR'C'D eu latura de 1D m, Be AN MN : ni loeete turd [48]. (40). (C°D" i [84 LAU Tianp se Ter $0 minute Se aeord | punet ain of | etircoa PUAN + Atarus PERIAN + 8 4) Athi ch punctele M,N. 47, 8" sinc eaplanare. MATEMATICA Clasas il-a —1 10. oe ) Daet proprietaraé ineaperitdoreste st vopseased wiunghiuile MN si te alb gi resiul porstilor (inelusiy pedetua gi wvanulp ce albastr, il vopsea liebe 54 curnpere sind e9 felosesie F ky de vopses la 1 nv? Dac’ 1 ky de wvopsea alba cosid 2 de lel, iar 1 kg de yopsea altastrd vaste 24 deh cleterminuti surna de hat necesard achizijionait vopseleler. Posen alavemnt. Aguiar 3. roprerim dou fee li deste wut poralelipiped cirepuun: a) Deserta Sntveaga destiguears By Dai df > 20'em, AB =a0 em gi BE=NOern. ated | dovermnsyi eee euburi eu mucha ole 21) ern inert acest paraleipined éveprunghic © aed AF =12 om, 44=18em gi BC=3 em, aunel deterinall mona Pini ce cuhuri de oeeesi muckie care oeupa tot spatiel parklelipipectlut dieptanghic (0 minisert ave forrna uni cub ABCDA’BC'D’ cu lature de 20 m. La intersection ugorulelor [AC] si [4°C] so fixes o Surst de lamina leyata prin cari de eele pawn value’ sle evan 9) Detecminayi lungimea ttala a eablurir foloste. by Avia e& dach in sor sunt Iisa 53 aboare 28 de Dutur. alunei ex ori mah miva, de ehte Ini amem nevoie pentra a vopsi gi ptfile neseuperite fei se wopse ex. Dawa apa ce & fost fe vapsen 117. Fontan pavarca nei striate folosese picts parulelipipedice cu lumuamen dle 14 em, lajimea de 8 em si fadlimea de 6 cr. 1a) Ce suprafaja se poste pete ev LOA de pictre? {) Daca sitada are gies de @ m, ue cite pietre este mud vie pM pa ae WH kin de strat 118, Interiorul wui eub ABCDA‘S'C°AT este Hormat din ogtingi perivere. Pein yeti A plitrende in interior o rvzf de lanvind, se cfleetd pe Tafa HCE gi ajmue is vise [AT Se slg €&: muchia cubulu are luiyginea de 4m, w Determinati langionex duel poreurs de raza de I In bp) Dacd M este puetal im gare raza de Renind ating Sapa (ACC, cake si triunghiula 844” 19,0 cumerd are forma amui eub 48CDATCR™ ew Jungimes muchiei de 4 sn, Punctol #reprezinta pevita tina ptinjen sitwat pe perctele BB'A la distant de aga de mebitle AB si BB’, (arin puntul 44 se x14 ‘6 nased situalé pe percisle CAD" la clstanya de Fae Tos de muchille DD" yi DC 4a) Aralafi ch punctele P. BY Afgi D sunt coplanare, 2) Calevlayi angie siumnulal celai mai seun pe vare lrebuie sh meargh phianjenul penn a ajuoye Ia museca A b (Ge considera ef pianjenul se pewte deplasy doar pe poaes, pes pont gi pe asa 9 Determinagi distant de lu muse la muchia BA 20.0 camera fiigorided in forma de paraletipiped ute cubbice, fineace avid fara de din. Ts 0 rn ceamere! Higodliew este aeopertl complet de 7 poet Sori mai mare deat fnatgimes unui pach. 1a) Determinayi aria supratejet podeled Incipert 5) Un perete lateral al cemerel se vopseste. Dae pent T din? se Folosese 1) ae de vonsea, iar | Ie vopsea spevials cost 16 lei detemaimigi eit esta vopsete vuiiznia, D2. Lat un concurs, un alpinist webuie SI wsealilere ui blue de piste i ves prism deeapid eu bree potiet. AACOABCD' A= Fm. BL Sangh Pn W) Alpinistal Uebuie $9 ange alin | Tf urversdnd Mveace Lay leva blogulu. Care este leagénica miniant « eeseul? 1p Daca iraseu! alpinistoln) Inkalneste muehie [4°] yi [CC tu prmetete i « Seierminati lungimen sezmertule (PQ). e pling eu pach i9% oui re ele Poaleatst Inghimey eamergi este de t = a = (cea FIANU + aus PERIANU + Bamiteu SAVULESCD M Tema 2.4% Droblema pantry performanta scolara si olimpiade Se cad ul seidepte Be, Op Os pe cares iu punetle A,B, Cate Dia Ob 30 Oe he al uh bree) ata ee gente a peoiectici punctalui pe planul (22C). Iman plan seat un eete de cane O 6 cuzn Q4 = 6 a Po core 5 ia punetele yi C asl ea 48] = [AC] gi BC = 66h em. Sepisenal {OV ete permendiulte pe pmula i ¥ = BY = CY = 12 ems Seneca ou proisia. [alt pe Vt A an iui ACD, Se considera chu ABCDB'C'D! ip care AD AD = {0} gi punesul 18 sic mijieen muchisi 4, Demouserati ef MOL OBR: By MO L(#C'D) Fie paral asifel inet uinul ICD. Se constnuiese drepiefe paralele AM gi pnctele A751 ¥ sunt ia acelagi semiplan determinat cle (BC) st dad 4a} Avital ¢8 planele (448) 91 (CDAD nw sust paralele. By Deed ABCD =. BY Od = VP}. MDMd = {Q} gl AB = 20 om calculi PQ. Pe plomal pfirstulul ABCD, cu 48 = 3 em, de acezagi parte a planus, se ‘onstruiese perpendiculacle 2. ~ 6 cm, CC"= 8 em gi B'D = 4 ern, Determinali punctul 4’ aM pe perpendiculara in. pe plant pswatului, asifel meat 4°, BY. C" $1 O38 ie eoplanace Se vonsier piramnida palrulaterd regula “ABCD cu bwza ABCD $ un plan oc ABC) cave inersocieaca seementele (VA), (7B), FC} 51 (PD) in punetele MN, Po NA OD wep? ar or P si cespoctiv @. Arata Pe plinul drepiunghiului 1800 (48 > BC) se sidiet peapendicwlaen $4, Dac sie mje! segment [SC] se cere: Demonsiati ck tiuoghind DEB est isasce >) Apitaliea BD & SC ¢) Dae Meste myjleen) Ii [BE|, Sf a BO 8 A || 840} EN), lar BDAN = (P} arataé Se considers 4, 8, Ciel priete pe an cere de raza + astfel neat sriunghiul 48 geste echilaroal. Pe plana coreulus se ridied perpendicularele 4M BM, CP, i acelagsemspatiu delinital de planul (ABC), astel incat AM = 4a, BAT = CP onde a 0. Artiayé ca devapia de interseetc dntae planele (4B) yi CLEP] este tongents la cere Se consided punetele nevnplanare i, 2. C gi 1. Fis E © CD astfeline€t sume 11° EB sa fhe main si # interseeyia bisectoaved unghiulu [EB evs dreapia 12 Alay’ ck BF LCD, Un paraletipiped dreptunphic aBCDA'B'CD" ace baza ABCD poeeat, ian AF si 8 sunt picioarcte perpendiculareloc duse din 4’ si D pe diaganala BO’ Demonsiali (oh ADNB" est pacalelogram dacs g1 nunnai dacs: ABCDA'B'CD' este en Fie paralelipipedul dreptunghic 42CDMNPO gi punctele Ee(BN), Fe{DO) asctel net suma Ak-+AF+PESPE este mini’, Ardtagi ef BFLAP deci si nomai dsc ABCDMNPO exte prism rogelat i Jn figora alsturata of Lae, || AP = 3.0m gi AB fom, Prin pusietl P reece 9 dreupts care fatenpa pled a {in C si planul B te, asetel ineat AC 4 cm. Calculati perimerrul APBD. | maremarica ciao iro -1 CAPITOLUL ~ PROIE Tema 2.1, Prolectil de puncte, segmente si drepte pe Un plan ‘Tema 3.2. Ungbiul unei drepte ew un plan, Lungintea praiectiei unui segment Tema 3.3. Teorema celor tres perpendiculare este de evatuare Psi pointe portofotin? Tualivities® Tem 3.4, Unghi diedru, Plane perpendiculane Tema 3.8.Calculul unor distante si mfisuz de unghsuti pe fefele saw in inter Feste ile evaluare ul coxpurilor studiate Fine emia porvtpeiies a {6 Probleme cu caracter aplicativ 7. Probleme pentru performanga scalarel gi olimpiade Fema ss Dach ponetl Beste eats , shia, ae pitts psu se panihacene BF vito perpen dust dis puntl Mpeplanel an fg Bea (ops aes {38° Le, anc nom 23, Dea de fvmso ates Dat pital Aapaiae plant anc 7 pwoiecie poncuie # fe pacul @ cowelde cu puncaul't: J Ly det dec, atunci A= prt i Srsiestis sant! “egment: Poiegtia segmental! [48] a ve planul este segmenul [4°87 ale ef extemiat sink 7 proiectile pe plenl a sleestenitilorsegmentla dat $7 Nott: 4B] =E94 4 mM) =L4'2T eee 4 : Daca deapia 4B este perpendiculara pe G Hal ose pregnant [a ppd wie reduce ia ua punet As Mrolsta unei drpte dpe plant este dicapta deveined pret pe pat 7 {ca dovt pute dite ale dpe 4 Notaen: pat lanl a stuns! proieetia dleptei pe planol a este un a proicetia dieptei pe planul est 0 punt 2 Ded AO.La si AOra= (94), tone 4} =pr,40. de Gbsuewasie Daca dreapta of este perpendicvlers pe a / detente. Te general, proiectia figurii geomeitice plane F pe planul o este figura geometriek F' format din reuniunea proieepitar punctelor figurli F pe planvl freemen casa Inirces FLAN + a a J Duneiele 4 gi # sunt de aceeasi parts 2 planula) be Pe cubul BC''CD' (eal ysis). Nomi 4) phe BE phar ©) pr. A) prow lCS ed pry Pet ues AC” HI PPeiie ABS ADD ye AADC B preyneot BOD Ih figura alntura, HadCD este e pitsimida patsulaters rewulatd Th eare € este proiceyia ortogenala s waefolu pe planal bazei, Precizail OP yrlVI? 8) Phal MP unde Me BON AP Hasuead prow Ale si pried PMC iar punesele a? gl BY sunt projectile punetelor t gi 8 pe planel er histificat a4) Duca Bie akungt 4°B"|| AB: Arita ch 48's ABs 1) Dac 4850 = 10}. atunel punetele 4. # yi @ sunt caliniars Puncicle A yi @ sunk de a parte si ve abt a planotui a iar punctele at gi sunt proieetile puneteiar 4 gi B pe plan! ar. Dach 4a = 10} arate 4p 8 08 08 BR OB OB Un scumient [AR] se pinicetear’ pe un plan ar dupa seunrental [48], unde A” sh BY sunt provectile pinctelor 4. respectiv & pe plouml a. Dach AB = on. AA's 6 con, iar BB! = 9.9, alal longiniey proivctiei [472 Phineteleevfiniane A. 6 54 C sont de aceeasi parte a plant @ far ponetele 4". 1° $1 C% sunt proiecyile punctelor 1 '§) respeativ € ps planul 1) Justices ea punesele 4, 8" si C” sunt coliniare, 4B BC O81 B sum coliniare: a) Punerele ae by Aviag ok ep Pack 484 om, AC Ibem gi BC = Sem, ps Arial oh Se o caleulays 8" mente) AC se proieste pe planl @ dupa seymemul 4°C* (veri figura 4 altturatay, Se stie wi B = [40] asthe fete We ped gia ap Calcul 2°C"; ‘| ening oR ‘Trapezul ABCD are numai haze race AB conginuta jn plana! a, ier AB=2-DC . Distents de ln / punctul C la plomui a este de 36 em. Notins Zo [O}= ACO EP si 0 =p,,0 e) Determinali vatoarea rapsrtubai by Caleulaus 00" 1 « Triunghiu) AC este dreptunghic it &. 6 Tei hiul PH ie aF njlocul segmental [BC 38 @ een ul ee hihi 8¢ Ps AF" si G sunt praectile punctelor AF gi vespeetiv Gps plunst 6 ra 4) Bunctul M7” este mijlooul» jk 6" se pruiecten pe plowul er ehapa wi cgmentult (PC 16; Punctul G es eentvul de yreutale al tiumghiaiui 6° Poy Pawulaterul 4.8°C'D" este proieetia pe planed @ s povalslagramului ABCD. bie 10} = ACABD 81 OF mijlocul segoentulii [4°C"). Arata a = ,0; —— &) Pairulaieral 4° Unghie drope AO# ane Fura [04 pavalsla cv plasul a Ponetele f, 0 $i 8 se proiectesza pe planar th punciele 7, O' sirespectiv WU Sigi et w £O'-LO8 by mt FOR) = 90 fie Figua ofStwrats ABCD este pawal, tar 86" este proieetia fui pe planal 2. Arata este paralslagran. a) Dock BE}, mre! paurviwerad °C" este dreptunghis = bP Dred BE|| er wlune! patrulawral 0°49" ste torn, . i Fie pitratu) ABCD. in vinturile C si B se cilia G24 _ peipeadicalarcke CAF respectiv ON pe plant pateatuli, Aratuyy <8 preleetile nnijleacelor segmentelor [4M] $i (2 pe plunul (480) esineid, Unghiul 408 are lature (OA parsleld cu planut c . Puneisle 4, 0 yi B se procctewzt pe planula Tn punctele .t', 0" sirespeetiv BY a Dach [OB lee .arttati ck FOR = IOP by Bact [OB , @ comparati masuvile unghiucilor AOD si 40°H" tw easily By Unghiul 408 este cbure: di) Unghiul 408 este aseuti re cata [18] fees Cea Fis C proicetia puncte Cpe plana a. Ce AB cm si C= ec em, Anat et Tn planed ee Seslie ok 4B =3em, BC apiriunghia) ABC’ este dreprunyliies 2) 4. oy ol ACB) < ml FCB) by oN BAC) > ml BAC) ‘Triunyhinl Isoscel ABC cu AB =AC are vist 4 Cx stat n paola, Dremia ACestepakticut, SO 5 “ pr.2 = #" demonstrate a) Veiunghiul ABC” este isoscel DP a BAC) > m(ABAC] 3 2 2 ANU » Maris PERIANU + Dumitiy SAVULESCU Tansces Triunghial isoseel RC are bn26 [BCT intelush in plana a iar ponctul a este provectia punctului et pe planul a, a € FC. Demaristeati ed ) Vriunghiul 4°8C este isoscels 4) mi BAT) > m( BAC) Trion Fsosee] MC ace laura BC inclust 3a plana er iar proieetia lui A pe phinul a este puueiel Af Se slic ef wi BARC) = 90% Mc =1heu Cateulaté perimettul rungbiuie; ABC. Pe yervul de gentry (2 yi nz #= 10 em se consider’ punstele 4, B, C. Pumetul 2Y se proieeieaza pe planal cercului im punctul 0. Se sti cf OM som. a Anat 6 Abs by Cateulati cos MAO) WH = nie Se considers texsedral ABCD In care AB=AC= AD, sat a este proiectia vanlulut 4 pe playul (BCD), Arttati e& punctul a’ este eonteul eereulvi circomseris trimgbilut BCD. Piraimids ABCD ate ca baza patrulateralinscciptbil ABCD cu eeniel Q. Stine C8 AFR =IE AVI arial C8 O= Phases Se consider pirsmida paurulaters cegulatn ¥ABCD eu been ABCE In care Haz AC = by? om, 4a Caleulatilungimica protectiet machiel #4 pe planul bazet 2) Determiiagi masts unghiul format de dvepiele Pt si ACS 2) Caleola lungimea penveeties rmclies M4 pe planul { 72D). fb oe de saute tele Pieaniida patrulatera regulals VABCD are apetem [VA], M€ BC. Punciul O cel eontrul Daze) piranidei, Dac VM = 4# = 12em, eal 4a} indiinnes. prams ‘By aio seotini dingonale ¢) soma lungimilor protegiitor twturor nwehiilor pe planul bazed ‘Triunghiel echisteral ABC este inchus fh plana a este prosectiairianahiuiu DAC pe plu! a. Fie & milocul segmental LAC. Se alle of AR = [Zein si AD =8en ay Atbay ob BCL (DEA): By Atti OC); 6) Ala ania tinh DBC. 1) Arty ck A = cos BEA) Hie rwmbol ABCD de vent O gi AC = 8 em. in punetele 4 si C se tidied perpenticularele 443i CC" pe planul (48C), de aceeagi parte & plana a) Arata ed Ot cox TO) = OC eos OC); 67 Dah OL CO Ad’ > Fem si CC"™ x calcula. Unghial dzeptei ef cu planut a este unghiul format de dreapta ¢ en provectia ei pe planul a pia atuvel nfdiary = michal We Dact dca sau eloratuncl m(dcy= 0" Dact der alunei ontdoer) [4B] tunel Se considera un segment [487] si 9 érespth d, Duck [2°8" AB = AB. cost ABA) veo espe. Dank AB [dt atonci 4'B"= AB iar dack AB Lef,atunci 4'8"=9 Se consider un segment [48] sium plan a Dacd [1 ae Dacd 48 j}cc,atunet 48" v, [48] -atune’ AB cox AB. AB iat duc AB La, atunci 4°68" Un segment 4B = 10 em se proiecteazt pe planul a dupé segiemtul [4'3% Catowinti 4°" daens a) ea Aa) = 80°; 6) abe) o) a TB.) =, = Proviaai pozitia deeptet 4B tn rapart eu planul a daca ©) 1h ABB ) oi Al Reprezentati un plan ee si o deen ldreapls d se consideri punetul 4 astlel Sneat O4 = pepe pe cm pl 0. Pe can jar in planul @ se consider un put B.Determinati mA.) acd @) 4e=33 Abas; 9 ABET ‘Desenati cubut din figura aliuurata si deterinas 2) BRAD & cost AC LAB) [meroumetcin ‘Cubul io tiga atiuratd ace 2 =e, Papin y . © prism! putnalaese regula ABCDADCD! ane sd =N cam, Suid cd unghie Voom de divapin -1C* eu plaral GEC ese cat ot 60%, ala angie cick fect Mochi late fa] tnchiviai de pinwnidhi patmbters regia ABCD'B'CD’ iormcnat cu planul baaet mac un ungh ca risus de 4°. wy Avital ci ACC by Daca Ad’ = 87 om, sleuitiiniltimenirunchiull de picuwid 8 Desenati un cob ABCDA'S'CD" . Dock AB=4 cin, espriensi lungimnea segmental care repeezinléproeetia segment [49] pe adept AB; A) éreapta CC": ep dreapea aC: a) dveapta BE” 8 Nov eu 0 eomrul bezel ABCD & prismel patwlaterereguate JCD" tomsi Co"= 73 em 4 Calcolé migra unghivtul format de éeapta OC” cu plavul bor 4) Demmonsteai ca teunghiul 10C" este deeptunghiex 6) Caleulat ongiinen proiectie) segment [4a] pe denpia 40. Se-euncse AB ee 10 Desens in tetraedru regular 4HCD Im care 4B = Gem, 1a) Punoi in evidengs fn desen proieefa punctului pe plawal (BCD), ) Cateulati iangimea proiectiei seementalyi [42] pe pianul (BCD), ©) Coleulagi ox, ABTBCD)) 4) Cale lungimen proiveyil seymencalui (08) pe dreapra CD. Triunghial echifstsral 8C, cu 42 —3 em, se proieeteuzh pe plane! a dup triunghinl deeplinghie 475 4) Precizati unghiul drept al triunghialul «°BC : 6) Determinati wa Fine). ¢ Triunghiul dreptunghic AC, BC=6 em, se proivetea2d pe plaoul @ dup iunghial isoseel LBC cu (BPC) = 120° 1 Precizabi unghiul drepe al tiunghiuhii ABC, B) Aritatica AB = 4 6) Caleulasi cost FEE) ; Calcalati d( 4a) +2 Tn vénful 4 al picratulul ABCD cu AB = 12em se cdliod perpendiculara 42M pe planul patatulei, 4f =12V2 em. Cateulay a CM ABCY) ¢ 4) cos, DMAABC) 4 in vate gt ale patraile) ABCD ds gomteu Gye ridied pe pana wees de news parte, perpendicul ele fF espeetly BA Se tie ct -I =7 em. ald’ = 2am, HB =Yem. 4) Caiculat mrastra anghinl Fema ce dean 16" bp rains et ine PAE AC a 8 evasion 128 [ [0 5. Pe planul rombului ACA se rset perpenutcurt AS. Duet 1 gi mA) = 6. cole ny eos SETAC} m{OB(ABCD) unde Oeste milocul seurnemtula [SC 45. Punewu) O este incrscetin diagonalelor roinbalsi ACD Sy came Me® 5 Pe, wnt THC) =120" in pucicle Osi Cs ic, de webey pate pln Jtpendvularcle ON Ci! pe plu! mab Se ghee Cava 2sem Denna c8 puncte 4, Ms etnias 19 Decent wl ATE oy Cael sl WOT TRC 1 4CD ave wate mci oe gate $s PO Naa imide PAC ete cela. 1p Decerminas nh FAABOD) 6 Calelg angie. poeais ss reapis cP. 1 Deiat i usiicaivloarea de adevtr a propo “san sega congrusnie. Foe area pe plano ake seamen [A] $1 (sunt sega Tone kegel i 1D st pals wt spl [4] 3 EDI mat org ake! pos spp ve dep eid ee um sezment omg ese FS Ce sccm TC] ye dupa Baek mals souneticey Li) 9b [CL pe dey oe Eanes seamontle {IAL S}ECDL sn eemecont= ts: Desf hal ABCDAECD" Dash 1 =! Mea sgmerghi soe vedi rota se 2 (ADT peateapa Bes) (BI) pe eeaapa AD": © [40"] pecteapa OC: Wd) [AP] pesoeia B. 20, Pe planal dreplunybilui ABCD se egies papenciews it Se sie oh AB «Bex, mB LARC =P gi OAL) = 85° 1) Caleta ungives poets segment [C1 pe dsp 1. by Caletay cost ACH): Casula aC HAC) tes piles [21] pe le lor pe plant sxprimal fl sfremmanencanon

You might also like